Problem Sets

Published on June 2016 | Categories: Documents | Downloads: 72 | Comments: 0 | Views: 720
of 72
Download PDF   Embed   Report

Comments

Content

******************************************************************** Please Note: The answers appears at the end of this document ********************************************************************

******************************************************************** Start of The Questions Section ******************************************************************** Chapter 2 - Question# 1 - In a civil assault case, the plaintiff introduces a blood-stained golf club that she claims the defendant used to beat her. The club is: A. B. C. D. Demonstrative evidence. Real evidence. Oral eyewitness testimony. Stipulated evidence.

************************** Next Question**************************** Chapter 2 - Question# 2 - Defendant is being prosecuted for possessing narcotics with the intent to sell. The prosecutor introduces a plastic bag of powdered sugar and asks the police detective on the witness stand to use the sugar to show how the defendant packaged the narcotics. The bag of sugar is: A. B. C. D. Eyewitness evidence. Expert evidence. Real evidence. Demonstrative evidence.

************************** Next Question****************************

Chapter 2 - Question# 3 - A small software company sues three larger competitors, claiming that they violated price-fixing laws by intentionally agreeing to sell products at the same price. The small company introduces pricelists distributed by the three competitors. The three pricelists each advertise fifty different products at varying costs; each product, however, carries the same price on all the three lists. The three pricelists are: A. Real evidence that provides direct evidence of an intentional agreement to fix prices. B. Demonstrative evidence that provides direct evidence of an intentional agreement to fix prices. C. Real evidence that provides circumstantial evidence of an intentional agreement to fix prices. D. Demonstrative evidence that provides circumstantial evidence of an intentional agreement to fix prices.

************************** Next Question**************************** Chapter 5 - Question# 1 - On direct examination, the prosecutor asks a witness for information that would violate the attorney-client privilege. Before defense counsel can interject, the witness responds. What type of motion should defense counsel make? A. B. C. D. A motion in limine. An objection. A motion to strike. An exception.

************************** Next Question****************************

Chapter 5 - Question# 2 - An injured taxi passenger files a negligence claim against both the taxi driver and company. At trial, the passenger offers evidence that is admissible against the driver but not the company. The trial judge admits the evidence, and the company's lawyer requests a limiting instruction. Which of the following is correct? A. The judge instruction. B. The judge C. The judge D. The judge do more harm should require an offer of proof before giving the must give the instruction. has discretion to give the instruction. should not give the instruction because these instructions than good.

************************** Next Question****************************

Chapter 5 - Question# 3 - With respect to most evidentiary errors, an appellate court will reverse the trial court's judgment only if: A. The trial judge abused her discretion. B. The error affects a party's substantial right. C. The trial judge ruled with specificity. D. The trial judge abused her discretion and the error affected a party's substantial right. ************************** Next Question****************************

Chapter 6 - Question# 1 - Henry Handler was driving south on High Street. As he was approaching the light at 24th and High, Wally Witness, who was driving in the car next to Henry, observed Henry talking on his cell phone, drinking coffee, and looking down at his CD player in the car. Two minutes later, at 12th and High, Henry turned right on green, accidentally striking Paula Peddler on her bike.

Paula sues Harry for negligence. Paula's attorney wants to call Wally to testify as to Henry's behavior at 24th and High. Is this evidence relevant? A. No, because Wally observed Henry two minutes before the accident. B. No, because there is no evidence that the behavior Wally observed caused the accident. C. Yes, because Henry's actions while driving at 24th and High tend to prove, however slightly, that Henry was not paying close attention to his driving two minutes later. D. Yes. Because only two minutes elapsed between the observation and the accident, it is more likely than not that Henry was still talking on his cell phone and drinking coffee at the time of his accident.

************************** Next Question****************************

Chapter 6 - Question# 2 - Dozens of women are suing Farnsworth Pharmaceuticals, claiming that its new anti-depressant drug Serenity caused birth defects in their children. Under the applicable state law, Farnsworth is strictly liable for any injuries caused by products that it manufactured, but it claims that the chemicals in the drug could not possibly have been the cause of the birth defects. Which of the following pieces of evidence would be relevant to the case? A. The plaintiffs offer evidence that the CEO of Farnsworth is a member of a White Supremacist group and attended monthly meetings which included a pledge to ³protect and advance the interests of the Aryan race.´ B. The plaintiffs offer evidence that Serenity does not in fact help treat depression. C. The plaintiffs offer evidence that Farnsworth did not conduct any independent tests of the drug before putting it on the market. D. None of the above.

************************** Next Question****************************

Chapter 7 - Question# 1 - Lou is accused of sexually assaulting his coworker Mary during the lunch break at their workplace. Mary called the police immediately after the incident. The police arrived at the front door of the office suite ten minutes after Mary's call and one of the secretaries called out: ³Hey, look, the police are here!´ A minute later, the police entered Lou's office. The office appeared to be empty, but the police searched the office and found Lou curled up under his desk. At trial, the prosecution calls the arresting officer to testify that Lou was hiding under his desk when he was arrested. The defense attorney objects, arguing that there are a number of possible explanations

consistent with innocence that explain why Lou might have been sitting under his desk, and that even if he were hiding it certainly does not prove that Lou was guilty, only that he didn¶t want to be arrested. What is the proper analysis? A. The evidence is irrelevant and therefore inadmissible. B. The evidence is relevant but probably barred by Rule 403. C. The evidence is relevant, and probably not barred by Rule 403. D. The evidence is only admissible if the defendant takes the stand and testifies that he never hid from the police.

************************** Next Question**************************** Chapter 7 - Question# 2 - Louie De Palma is accused of killing Latka, one of his employees, by hitting him on the head with a blunt heavy object. No murder weapon is ever recovered, so the prosecutor seeks to introduce into evidence a tire iron that was found in the garage where De Palma works. There is no evidence that DePalma ever possessed this tire iron, and no evidence that a tire iron was the murder weapon (though it might have been). The prosecution's tire iron is: A. B. C. D. Irrelevant and therefore inadmissible. Relevant but probably barred under Rule 403. Relevant but probably barred because it is circumstantial evidence. Definitely admissible.

************************** Next Question****************************

Chapter 7 - Question# 3 - Jenny, a school principal in Chicago, has been sued for employment discrimination. The plaintiffs claim that she has promoted white teachers ahead of more qualified non-white teachers. The plaintiffs' attorney wants to admit evidence that Jenny's husband is active in a white supremacist group. Is this evidence admissible? A. Definitely not. Her husband's involvement has no bearing on her actions at the workplace and this evidence will unfairly portray her as a racist. B. Perhaps, but only if the trial judge concludes that the probative value of the evidence as to proving Jenny's racial bias is not substantially outweighed by its unfair prejudice to Jenny. C. Perhaps, but only if the trial judge concludes that the evidence does not unfairly prejudice Jenny. D. Yes, because her husband's involvement in a racist organization makes it at least a little more likely that she is biased against non-white teachers and would base her promotion decisions based on that bias.

************************** Next Question****************************

Chapter 9 - Question# 1 - Leon purchased a Periladder brand telescoping ladder in May 2000. In September 2000 the ladder collapsed while Leon was on it, causing Leon serious injury. After recovering, Leon recommended Periladder to his neighbor Carl, whom he deeply loathed. Carl purchased the same model of telescoping ladder from Periladder in November 2001 and was similarly injured in January 2002. Leon and Carl sued Periladder separately, each bringing a strict liability and negligence claim. In December 2001, Periladder modified the design of their telescoping ladder and began affixing warning labels to them. At trial, Leon seeks to admit the change in the design for the purpose of showing the original was unreasonably dangerous and Carl seeks to admit the new warning labels for the purpose of showing that there was a need for a warning. Periladder raised a Rule 407 objection in both cases. Periladder's objection will most likely succeed (thus precluding the evidence) in: A. B. C. D. Leon's case. Carl's case. Neither case. Both cases.

************************** Next Question****************************

Chapter 9 - Question# 2 - After having most of their products removed from nationally-recognized hardware stores, Periladder began selling their telescoping ladders on the Home Consumers Network (³HCN´). Erin McCain, an employee of HCN, was demonstrating the telescoping ladder during a November 2001 live broadcast when it collapsed, seriously injuring her. HCN had to run nine more shows in which they needed to demonstrate the ladder, and so in December 2001 HCN engineers reinforced the ladder so that the product could be safely demonstrated by other employees. Also in December 2001, Periladder made its own modifications to the design of the ladder. Erin sued Periladder for negligent product design. At trial, Erin attempted to introduce evidence of HCN¶s modification to show that the product was unreasonably dangerous before the accident, and also evidence of Periladder¶s subsequent modifications in order to prove that a safer design was feasible. Periladder did not contest feasibility in the case. Periladder raised a Rule 407 objection to the admission of both remedial measures. Periladder¶s Rule 407 objection will most likely succeed (thus precluding the evidence) with regard to:

A. HCN¶s modification.

B. Periladder¶s modification. C. Both modifications. D. Neither modification.

************************** Next Question**************************** Chapter 9 - Question# 3 - While shopping at House Depot, Shannon Burke noticed a display of Greencut brand gas-powered trimmers piled in a stack. When she approached the display, the stack of trimmers toppled on top of her, causing her serious injury. Shannon sued House Depot for her injuries. During the defense case, House Depot presented evidence that a Greencut sales representative had placed the display there only a few minutes before the accident and had done so without House Depot¶s consent. In rebuttal, Shannon¶s lawyer attempted to enter evidence that, following the collapse, House Depot gave one of its employees a written warning for not properly securing the trimmer display. House Depot raised a Rule 407 objection. What is the proper ruling?

A. Overruled, but House Depot gets a limiting instruction when the evidence is admitted. B. Overruled, and there need not be any limiting instruction when the evidence is admitted. C. Sustained, because a written warning does not qualify as a ³remedial measure.´ D. Sustained, because the evidence is only relevant to prove that House Depot was at fault. ************************** Next Question****************************

Chapter 10 - Question# 1 - Mr. Ingalls made an oral contract to sell one hundred cords of wood to Dr. Baker. When the wood was delivered, Dr. Baker found that it was inferior pine wood, not the oak that he believed they had agreed upon. He immediately called Ingalls to complain, and Ingalls responded: ³O.K., O.K., I¶ll send you oak; just don¶t sue me over it.´ Ingalls never delivered the oak he promised, and was sued by Baker for breach of contract. At trial, Baker wishes to admit Ingalls¶ promise to send him the oak. Will this be admissible?

A. B. C. D.

No, because of Rule 407 No, because the statement is irrelevant to the contract dispute. No, because of Rule 408. Yes.

************************** Next Question****************************

Chapter 10 - Question# 2 - Michael Cameron, the owner of a successful restaurant chain, gave all of his restaurant managers and head chefs generous employment contracts which offered long-term employees health benefits 'for life.' Cameron's restaurants began to fail, however, and it was soon apparent that they would have to close. Cameron met with all of his employees receiving benefits, told them about the closings, and offered them a lump sum payment in lieu of their health benefits. All of the employees declined. After closing the restaurants, Cameron stopped paying health benefits, explaining that continued operation of the restaurants was an implicit qualification in the contract. During trial for breach of contract, the employees attempted to introduce evidence of the lump sum offer to prove that Cameron had agreed with their interpretation of the contract and therefore Cameron and the employees had a "meeting of the minds" that the lifetime health benefits would continue even if the store closed. Cameron's lawyer raised an objection under Rule 408. Will Cameron's Rule 408 objection be overruled? A. No, because the offer was made during compromise negotiations and proves liability. B. Yes, because it impeaches Cameron. C. Yes, because there was no dispute at the time of the offer. D. Yes, because the evidence is not being offered to prove liability, the amount of a claim, or to impeach a witness.

************************** Next Question**************************** Chapter 10 - Question# 3 - Tom McManus was fired from his job as a mechanical engineer for MLG Manufacturing and filed suit against his former company under the Age Discrimination in Employment Act. During litigation, Tom's old boss, Karen Leahy, asked to meet with him and Tom agreed. 'Listen Tom,' Karen said, 'maybe some of the higher-ups had a problem with your age but I never did. I support you in this lawsuit, but it looks like corporate management is determined to fight you to the end. In the meantime, I'm willing to make some calls to people I know at Acme Manufacturing--I'm sure I can get you a job there with a better salary than you ever got here." Tom replied, 'I don't like the work ethic at Acme--I want my old job back here.' Karen did not have the authorization to re-hire Tom, so he remained unemployed. At trial, Tom attempted to introduce evidence of Karen's statements and her offer in order to prove MLG intentionally discriminated against him. MLG raised a Rule 408 objection to the inclusion of the evidence. A. Overruled, because Karen and Tom were not in compromise negotiations

B. Overruled, because the negotiations were not in connection with a claim that was disputed. C. Overruled, because the statement cannot be used to prove liability. D. Sustained. ************************** Next Question****************************

Chapter 10 - Question# 4 - Steve Houston owned a national chain of art galleries specializing in high-end original paintings. Customers became concerned about the authenticity of Houston's merchandise when pieces entitled The Shriek, The Arnold Feeney Marriage, and Lunch Out On the Grass did not appear to be painted by artists they were attributed to. The Federal Trade Commission brought a civil suit against Houston, and after it was discovered that all of his paintings were fraudulent, Houston entered into a settlement agreement with the FTC. As part of the agreement, Houston had to pay fines, agree to stop selling forgeries, and sign a stipulation admitting to fraud. During the negotiations with the FTC, Houston stated, 'I knew the paintings were fake, but I figured the buyers knew also'they were getting them so cheaply.' As soon as his trouble with the FTC was over, however, Houston sold an 'original' painting, Cloudy Night, to a college student looking to add class to his dorm room. After discovering this, the government initiated criminal proceedings against Houston. While attempting to prove Houston's criminal intent, the government moved to introduce the statement made by Houston during the settlement negotiations with the FTC to prove that Houston knew his paintings were forgeries. Houston's lawyer raised a Rule 408 objection to the evidence. Houston's objection will likely be: A. Overruled, because the prior settlement agreement occurred before the current claim had arisen. B. Overruled, because the FTC is a public agency and was exercising its regulatory authority. C. Sustained, because the government is attempting to introduce evidence from a civil settlement into a criminal trial. D. Sustained, because the evidence is being offered for a prohibited use under Rule 408(a).

************************** Next Question****************************

Chapter 11 - Question# 1 - Numerous parents are suing Blocko Toys, Inc. in a class action lawsuit, claiming that its Junior Blocko set contained poisonous amounts of lead, which made their children ill. The parents want to admit evidence that (i) immediately after the children¶s illness became publicized, Blocko sent a letter to all of its retailers, telling them to pull all Junior Blocko sets from the shelves; and (ii) at the same time, Blocko sent a letter to all of the customers who had purchased Junior Blocko sets, stating: ³Enclosed please find a full refund for the

product.´ Along with the letter was a check which reimbursed the customer for the purchase price of the Junior Blocko set. What is the correct ruling?

A. B. C. D.

Neither (i) nor (ii) are admissible. (i) is admissible, but (ii) is not. (ii) is admissible, but (i) is not. Both (i) and (ii) are admissible.

************************** Next Question****************************

Chapter 11 - Question# 2 - David was working as an employee at McGraw Manufacturing when he his arm got caught in a machine on the assembly line. The foreman on the floor ran over to David, shutting down the machine and wrapping the mangled arm in a tourniquet. The foreman yelled for another employee to call 911, and told David: ³We¶ll get you to a hospital right away. We¶ll get that arm fixed up.´ David shook his head, saying: ³That arm¶s a goner, Ted²I don¶t have any insurance, so who¶s gonna pay to fix it up?´ The foreman replied: ³Don¶t you worry about that; McGraw will pay for everything. That¶s our policy, especially if one of our machines malfunctions like that.´ David ended up losing his arm. He sued McGraw, claiming that the injury was a result of their machine malfunctioning. McGraw claimed that their machine worked just fine, and that David negligently put his arm too close to it. David wants to admit the foreman¶s statement: ³Don¶t you worry about that; McGraw will pay for everything. That¶s our policy, especially if one of our machines malfunctions like that.´ McGraw objects, citing Rule 409. How should the judge rule?

A. The entire statement is inadmissible B. Part of the statement is admissible, but part of it is inadmissible. C. The entire statement is admissible. D. The entire statement is inadmissible only if McGraw did in fact pay for David's medical care.

************************** Next Question****************************

Chapter 11 - Question# 3 - Dr. Gordon was the anesthesiologist during a routine tonsillectomy when the patient suddenly went into cardiac arrest and died. As it turns out, the patient suffered from an unusual heart condition, which made him susceptible to cardiac arrest if a certain kind of anesthetic was used. Dr.Gordon claimed that she had no way of knowing about this unusual heart condition, since the patient had not told her about it and it did not appear anywhere in the patient¶s chart. The patient¶s family sued Dr. Gordon, claiming that the patient had in fact informed Gordon of the heart problem, and that the information had

been on the chart, but that Dr. Gordon had gone back and altered the chart after the patient¶s death in order to protect herself. During the trial, Dr. Gordon offered to admit that she is fully insured for medical malpractice, and that if she was indeed responsible for the death, the insurance company will pay for any and all damages. What is the proper ruling? A. The evidence is probably barred by Rule 411. B. The evidence is probably not barred by Rule 411 because it shows that Dr. Gordon had less of an incentive to alter the chart after the death. C. The evidence probably not barred by Rule 411 because it is being offered to show ownership or control. D. The evidence is probably not barred by Rule 411 because it is relevant to show bias on the part of Dr. Gordon.

************************** Next Question**************************** Chapter 12 - Question# 1 - Sergeant Joe Friday arrests Dutch under suspicion of committing an armed robbery. Friday reads Dutch his Miranda rights and Dutch states that he understands his rights and waives them. Dutch then gives Friday an alibi for the crime. Friday responds: 'Look, both you and I know that story isn't true. You really should be honest with me. Things will go better for you if you cooperate." Dutch then gives Friday a full confession. The prosecution later tries to admit the confession at Dutch's trial, and the defense attorney objects. The confession is: A. Irrelevant and therefore inadmissible. B. Inadmissible under Rule 410. C. Admissible only if Dutch takes the stand and testifies to facts inconsistent with the statement. D. Admissible.

************************** Next Question****************************

Chapter 12 - Question# 2 - Harry Sherman had too much to drink one night at a college party, and in a drunken rage, he threw a chair out of a dormitory window. The police were called and Sherman was arrested for criminal damaging. At arraignments, Sherman pled guilty to the charge. The next week, the college sued Sherman for the damage done to the window and the chair. At trial, the attorney for the college wants to admit Sherman¶s guilty plea for the incident. Sherman objects. What is the proper ruling?

A. The prior plea is barred by Rule 410.

B. The prior plea is barred by Rule 403, since it is of very little probative value and would unfairly prejudice the jury. C. The prior plea is not barred by Rule 410, because this is a civil case, and Rule 410 only applies to criminal cases. D. The prior plea is not barred by Rule 410, because it is a completed plea. ************************** Next Question****************************

Chapter 16 - Question# 1 - Zeke Channing was on trial for fraud. During trial, the prosecution asked police officer Thomas Wolfe if he had taken a statement from Scramando. Wolfe replied, ³No, I did not take any statement from him, and that¶s the sort of thing I would recall.´ The prosecutor asked if he had his investigative file with him, and Wolfe responded, ³No, I had it with me yesterday, but I didn¶t need it to refresh my memory so I just sent it back to the station. I don¶t keep up with paperwork that well, and I don¶t need a file to tell me that I didn¶t take a statement from Scramando.´ At that point, Channing¶s attorney requested that the investigative file be produced so he could inspect it and introduce it into evidence. The request of Channing¶s attorney will most likely be: A. Denied; Rule 612 only permits the adverse party to inspect and admit evidence that has been used to refresh a witness¶s memory for the purposes of testifying. B. Denied; Rule 612 only permits evidence that has physically been brought into court and used at trial to be admitted into evidence. C. Granted; Rule 612(2) expressly states that evidence used before testifying may also be admissible, therefore the investigative file would have to be produced. D. Granted; a witness cannot choose to rely on their own faulty memory when a written record of the event exists and is readily available. ************************** Next Question****************************

Chapter 16 - Question# 2 - After Michele Wilde attempted to sell cocaine to an undercover government agent, she was indicted on charges of drug possession with intent to distribute. At trial, Wilde¶s attorney asked DEA officer Lance Malfease if he had relied on any documents to help refresh his memory before trial. ³Sure,´ Malfease replied, ³I looked over quite a few investigators¶ reports and a few other documents.´ Wilde¶s attorney then invoked Rule 612 and requested to review the documents that Malfease used to refresh his memory and have them introduced into evidence. The trial judge denied this request, saying, ³I know what you¶re trying to do here, and I won¶t let you drag out this trial unnecessarily by requesting that all of these documents be admitted. There are other ways of impeaching this witness.´

Was the ruling correct? A. No; a criminal defendant has a right to see every document that the witness used to refresh his recollection, and the judge¶s refusal to order that the documents be turned over is an abuse of discretion. B. No; Rule 612 expressly states that if a witness uses a writing to refresh memory for the purposes of testifying that an adverse party is entitled to inspect it and introduce it into evidence. C. Yes; despite the fact that the officer relied on the documents to refresh his memory, the trial court judge may use his discretion to decide that producing the documents was unnecessary, and his refusal to do so was not an abuse of discretion. D. Yes; Rule 612 only applies to documents used to refresh recollection during trial.

************************** Next Question****************************

Chapter 16 - Question# 3 - Martin Rinker notified local law enforcement officers that he suspected his former friend and coworker, Greg Marietta, of drug trafficking. Rinker subsequently agreed to work with police in gathering evidence concerning Marietta¶s activities, and on several occasions Rinker recorded phone conversations with Marietta in which the two discussed a potential transaction. Marietta was indicted, and at trial the prosecution asked Rinker, ³Do you recall the details of the phone conversation that occurred between you and Marietta at 7:00 p.m. on the evening of June 5, 2005?´ When Rinker responded in the negative, the prosecution asked Rinker if hearing a portion of the audiotape from that conversation would help to refresh his memory. (The entire taped conversation was over half an hour long, and had already been provided to Marietta¶s attorney). Rinker responded that hearing the tape would indeed help refresh his memory, and he used headphones to privately listen to a one minute portion of the tape. After hearing the audiotape, Rinker testified about details of the conversation that were not on the tape. Marietta¶s attorney objected the use of the tape. This objection will most likely be: A. Sustained; Rule 612 refers explicitly to a ³writing´ that is used to refresh memory, therefore other forms of media are not permitted for this purpose. B. Sustained; a witness cannot claim that they are using a writing to refresh their memory and yet testify about matters that were not included in the writing. C. Overruled, because the witness did not testify about the material that he actually heard, so this was a legitimate use of Rule 612. D. Overruled; the fact that the audiotape is not a ³writing´ in the traditional sense is immaterial, and Rule 612 permits a witness to use such material in the way the prosecution did.

************************** Next Question****************************

Chapter 17 - Question# 1 - During a professional football game, Monus Lynch intentionally struck Holt Johnson in the back of the head with his forearm, causing Johnson serious injury. Johnson subsequently brought an action against Lynch and his football team for battery and negligence. During trial, the plaintiff introduced referee Ryan Chiaveroli as a witness. On direct examination, Chiavoroli testified that he did not see Lynch strike Johnson during the game in question. The plaintiff¶s attorney then asked Chiaveroli, ³Mr. Chiaveroli, do you usually wear glasses or corrective lenses?´ Chiaveroli answered yes. The plaintiff¶s attorney then asked: ³Were you wearing your glasses or corrective lenses on the day of the game in question?´ Mr. Lynch¶s attorney objected to the question. Most likely, the objection will be: A. Sustained, because Johnson cannot impeach a witness that he himself has called to the stand. B. Sustained, because a party cannot ask leading questions on direct. C. Overruled; a party generally cannot impeach his or her own witness, but a question about whether a witness was wearing glasses is not a question of impeachment. D. Overruled; this is an impeaching question, but parties are allowed to question their own witness¶s credibility.

************************** Next Question**************************** Chapter 20 - Question# 1 - Steve Kopel was charged with traveling in interstate commerce for the purpose of engaging in illicit sexual conduct with a minor. At trial, the prosecution called Catarina Gonzalez as a witness. Gonzalez testified that Kopel told her about his plan to have sexual relations with an underage girl. Kopel's attorney attempted to impeach Gonzalez with evidence that three years ago she was convicted in Florida of issuing a bad check for $75. In Florida, anyone who issues a check for less than $150 knowing that there are insufficient funds in their account is guilty of issuing a bad check, which is a misdemeanor. The prosecution objected to the inclusion of this evidence under Rule 609. The trial judge should: A. Sustain the objection. The crime is a misdemeanor that does not involve dishonesty evidence of the crime must be automatically excluded. B. Overrule the objection if the judge determines that the evidence passes the Rule 403 balancing test. C. Overrule the objection if the judge determines that the probative value outweighs the possibility of unfair prejudice to the prosecution. D. Overrule the objection, Under Rule 609(a)(2) the evidence of Ms. Gonzalez's crime is automatically admissible.

************************** Next Question****************************

Chapter 20 - Question# 2 - Jackson Lipscomb, who is thirty years old, was convicted of felony mail fraud eight years ago, but was pardoned by the governor three years later. Lipscomb has had a clean record ever since, with the exception of a misdemeanor for disorderly conduct. Subsequently, one of Lipscomb¶s former associates, Melissa Reed, was also charged with mail fraud, and the prosecutor called Lipscomb as a witness. At trial, Reed¶s attorney attempted to introduce evidence of Lipscomb¶s former crime to impeach him, and the prosecuting attorney objected. Most likely, the objection will be:

A. Sustained; a crime committed eight years ago is too distant to have any substantial probative value, therefore the prejudicial effect will significantly outweigh it and make the evidence inadmissible under Rule 403. B. Overruled; the crime in question is a felony involving dishonesty, therefore under Rule 609(a)(2) it is admissible regardless of the Rule 403 balancing test. C. Sustained; the witness has received a pardon and under Rule 609(c) evidence of that conviction is inadmissible. D. Overruled; because the witness has been convicted of a subsequent crime the pardon does not preclude the possibility of impeaching Lipscomb with this evidence.

************************** Next Question**************************** Chapter 20 - Question# 3 - Stanley Mancero is 18 years old and is currently on trial for arson. He testified in his own defense at trial, and the prosecution attempted to impeach him with evidence that he had been convicted of burglary two years ago. Mancero's attorney objected to admission of the evidence, and noted that there was a pending appeal from the prior conviction. The judge should: A. Sustain the objection, because evidence that the accused committed a crime as a juvenile is never admissible. B. Overrule the objection if the evidence would be admissible against an adult and the court finds that admission is necessary for a fair determination of the issue of guilt or innocence. C. Sustain the objection because an appeal of the conviction is still pending. D. Overrule the objection, since burglary is not a crime of falsity.

************************** Next Question****************************

Chapter 21 - Question# 1 - Shelly accused Graham of assault and battery. Shelly called Thaddeus to the stand, and Thaddeus testified that he saw bruises on Shelly¶s body the day after the alleged assault. On crossexamination, Graham elicited the fact that Thaddeus and Shelly were coworkers at the same company in order to show bias. Shelly now wants to call Erin to the stand. If allowed to testify, Erin will say that she has known Thaddeus for many years, and that he ³would never tell a lie.´ Graham¶s attorney objects to this testimony. The objection should be: A. Overruled; Erin¶s testimony is in response to an attack on Thaddeus¶s character for truthfulness, and only presented evidence of Thaddeus¶s reputation for having a truthful character. B. Sustained; Rule 608 only allows evidence of opinion or reputation for honesty, and Erin¶s comment fell outside of those bounds. C. Overruled; once a witness has testified, his or her character for truthfulness may be attacked or supported by either side. D. Sustained; Rule 608(2) requires that the character of the witness for truthfulness first be attacked, and that has not occurred.

************************** Next Question****************************

Chapter 21 - Question# 2 - After they were in a car accident, Brianna sued Chris for negligence. At trial, Chris called Devon as a witness. Devon¶s character for truthfulness was not questioned on crossexamination, but after her testimony Brianna called Lorraine to the stand. Lorraine will testify that she knew Devon well and that he had personally lied to her on three different occasions²twice about whether he was employed, and once about whether he owned a car. Chris objects to Lorraine¶s testimony. Lorraine¶s testimony is: A. Inadmissible because Devon¶s character was not bolstered or attacked on cross-examination, so Brianna was not allowed to call a witness to testify about his character. B. Inadmissible because the lies that Lorraine will testify to have nothing to do with whether or not he might lie on the stand. C. Inadmissible because Rule 608 only permits general reputation or opinion evidence, not testimony about specific dishonest acts. D. Admissible. ************************** Next Question****************************

Chapter 21 - Question# 3 - Brian Lindemann, a member of the city council of Summerfield, was charged with possession of cocaine with intent to distribute. The news shocked the small town, as Lindemann had a reputation in the community as an honest and moral citizen. Despite

this, Lindemann refused to take the stand and testify about any of the allegations. The prosecutor, who was concerned about what prejudicial effect Lindemann¶s reputation in the community might have on the jury, called Liz to the stand, who will testify that she works closely with Lindemann and believes ³he isn¶t a man you can trust.´ Lindemann¶s attorney objects to the testimony. The objection will be: A. Overruled; while evidence of a truthful character is admissible only after the character of the witness has been attacked, evidence of an untruthful character has no similar restriction. B. Sustained; Rule 608(a) applies only to witnesses, and Lindemann has not taken the stand. C. Overruled; Liz¶s testimony does not constitute character evidence, since she is speaking from personal experience. D. Sustained; Liz can only testify about specific instances where she has seen Lindemann say or do dishonest things; broad statements about whether he can be trusted are inadmissible.

************************** Next Question**************************** Chapter 22 - Question# 1 - Phineas Patrick, the chief financial officer of Drugs-n-Stuff pharmacies, is on trial for embezzlement. The prosecution calls Michael Fahrmore, an employee of the company, as a witness in order to testify about fraudulent financial documents he had uncovered. Later, Patrick testifies in his own defense and denies any wrongdoing; however, he does not specifically mention Fahrmore or the allegedly fraudulent document. On cross-examination the prosecutor asked Patrick, ³Were you aware that Michael Fahrmore is so honest, he was once cited in the Oldstown Weekly for returning an elderly gentleman¶s wallet that he found?´ The defense attorney objected to the question. Most likely, the objection will be: A. Overruled; the defense is not attempting to introduce extrinsic evidence, they are merely inquiring about such evidence, and this is permissible under Rule 608. B. Sustained; under Rule 608, specific instances of a fact witness¶s conduct (such as the returning of a wallet), cannot be inquired into on cross-examination of a fact witness. C. Overruled; Rule 608 permits a party to inquire, on cross-examination of a fact witness, about specific instances of the conduct of a fact witness for the purpose of providing evidence of that witness¶s character for truthfulness. D. Sustained; Rule 608 strictly prohibits the introduction of extrinsic evidence of specific instances of the conduct of a witness, for the purpose of attacking or supporting the witness¶s character for truthfulness.

************************** Next Question**************************** Chapter 22 - Question# 2 - Same fact pattern as above. Assume the defense called Gail Donovan as a character witness, and she testified that she had worked at Drugs-n-Stuff for many years knew that Michael Farhrmore very well, and that in her opinion Fahrmore was a very dishonest man. On cross-examination, the prosecutor asked Donovan: ³Did you know that Mr. Farhrmore gave generous annual donations to Amnesty International?´ The defense objected to the question. Most likely, the objection will be: A. Overruled; Rule 608(b) specifically allows such testimony from a character witness on cross-examination. B. Sustained; by mentioning that Fahrmore specifically donates to Amnesty International, the defense is attempting to introduce extrinsic evidence about a specific instance of conduct. Except for Rule 609¶s exception, such extrinsic evidence is inadmissible. C. Sustained; the question the prosecutor asked Donovan is about a habit of Mr. Fahrmore¶s²his practice of annually donating money to a human rights organization²and not about a ³specific instance´ of conduct. As such, it is impermissible under Rule 608. D. Sustained; the specific instance of conduct that the prosecutor inquired about was not probative of truthfulness or untruthfulness.

************************** Next Question**************************** Chapter 22 - Question# 3 - Assume the same fact pattern as above. After the defense attorney called Donovan to attack Fahrmore¶s character, the prosecutor called Marjorie Pergure in rebuttal. Pergure testified that she had known Fahrmore for many years and found him to be a very honest and forthright person. On cross-examination, the defense asked Pergure: ³Were you aware that, from 2002 to 2008, Fahrmore falsified his tax returns by claiming exemptions for charitable donations that he never made?´ Patrick¶s attorney objected to the question. What is the correct ruling in the case? A. Overruled; Rule 608(b) specifically allows such testimony from a character witness on cross-examination. B. Sustained; the event that is being inquired about here is not probative of truthfulness or untruthfulness. C. Overruled; evidence that the witness was convicted of a crime can only be proven by extrinsic evidence using Rule 609. D. Sustained; Pergure never made any statements about Fahrmore¶s tax returns and so this question is beyond the scope of her direct testimony.

************************** Next Question****************************

Chapter 28 - Question# 1 - Under which of the following conditions would reputation evidence of an individual¶s character NEVER be admitted? A. In a criminal assault case, to prove the peaceful nature of a defendant. B. In a torts case, to prove that a witness has a tendency to be untruthful. C. In a contract dispute, to prove that the defendant is known for breaking contracts. D. In a libel case, to prove that the plaintiff actually is a crook, as was alleged by the defendant in the statement that gave rise to the cause of action. ************************** Next Question****************************

Chapter 28 - Question# 2 - Andy Taylor is on trial for physically assaulting his 10-year old son Opie. The prosecution calls Aunt Bee, who testifies that two years ago she saw the defendant strike his nephew Danny in the head. The prosecutor also wishes to admit a certificate of conviction showing that Taylor pled guilty to the misdemeanors of assault and child endangerment in that prior case. How is the court most likely to rule?

A. Neither Aunt Bee¶s testimony nor the prior conviction are admissible. B. The prior conviction is admissible, but Aunt Bee¶s testimony is not. C. The prior conviction is admissible only if Taylor testifies, but Aunt Bee¶s testimony is not admissible under any circumstances. D. Both Aunt Bee¶s testimony and the prior conviction are admissible in the case-in-chief.

************************** Next Question****************************

Chapter 30 - Question# 1 - Michael and Gloria are getting divorced, and both of them are seeking sole custody of their two-year-old son. During the custody proceeding, Michael testifies that Gloria 'frequently flies into a rage' when their son is disobedient and that she once spanked their son with a hot curling iron. Gloria testifies that Michael once admitted to her that he had left their son alone in his car while he attended a graduate school class. If Michael and Gloria each object to the other spouse's evidence, what are the proper rulings? A. Admit all the above evidence to prove the character of Gloria and Michael. B. Admit all the above evidence, but only for impeachment purposes. C. Admit Michael's testimony about Gloria's behavior under Rule 404(b) to prove motive, but preclude Gloria's statement about Michael as inadmissible character evidence.

D. Preclude all the above evidence as inadmissible character evidence.

************************** Next Question**************************** Chapter 30 - Question# 2 - Ralph Kramden is on trial for assaulting his wife Alice; she suffered two broken ribs when Ralph's fist connected with her stomach. Ralph claims that he hit Alice accidentally while he was flexing his arm to improve his bowling swing. The prosecutor wants to call Ed Norton, one of Ralph¶s neighbors, who will testify that one week before the alleged assault, he was in the Kramdens' apartment and saw Ralph go up to Alice, shake his fist in her face, and say: ³One of these days I am going to strike you so hard you¶ll end up on the moon!´ The judge is most likely to rule that this testimony is: A. B. C. D. Inadmissible. Admissible to prove the defendant has a propensity to be violent. Admissible under Rule 404(b). Admissible to impeach the defendant if he testifies.

************************** Next Question****************************

Chapter 30 - Question# 3 - Bo Duke is on trial for stealing a Cadillac from Mr. Hogg. The prosecutor alleges that Duke stole the car by dressing up as a valet at Hogg's favorite restaurant and taking the keys from Hogg as though Duke intended to park the car. Duke has three prior convictions: (i) Misdemeanor drug possession from last year (Duke possessed marijuana). (ii) Felony grand theft auto from four years ago (Duke dressed up as a valet outside a restaurant and took the keys from the victim as though he intended to park the car). (iii) Misdemeanor fraud (Duke swindled a widow out of fifty dollars by soliciting money for a non-existent charitable organization). Which of these convictions (if any) are likely admissible against Duke? A. (ii) and (iii) in the case-in-chief, but not (i) under any circumstances. B. (ii) in the case-in-chief, and also (iii) if Duke testifies. C. (ii) and (iii) in the case-in-chief, and also (i) if Duke testifies. D. None in the case-in-chief, and only (iii) if Duke testifies.

************************** Next Question****************************

Chapter 31 - Question# 1 - An oil well exploded in Western Colorado, killing three workers. The well was owned by Denver-Carrington, a large multi-national oil company. The families of the workers are suing Denver-Carrington, claiming the company was negligent in maintaining the oil well. At trial, Denver-Carrington calls Davidson, the foreman of the crew that was working on the oil well before it exploded. Davidson testifies that he has been working as a foreman for five years, and part of his duties as a foreman include personally conducting a safety check on the oil pump every morning before the crew change. The safety check is an eight-step process, including inspection of all of the moving parts of the oil pump. Davidson will further testify that he has performed this check on every oil pump he has worked on every morning since he became a foreman, so he is sure he did it the morning of the accident, although he admits he has no specific recollection of that particular morning. The plaintiffs object to this testimony. The judge should:

A. Preclude the testimony, since Davidson has no specific recollection of the morning in question. B. Preclude the testimony, since Davidson is an employee of the company and therefore has an incentive to lie. C. Preclude the testimony under Rule 403. D. Admit the testimony. ************************** Next Question****************************

Chapter 33 - Question# 1 - Rob Petri is on trial for raping his wife Laura. The prosecution wishes to call Sally, a former co-worker of Rob¶s, who will testify that one year ago Rob attempted to rape her in the office after hours. No charges were ever filed in that case. The defense attorney objects to Sally¶s testimony. The defense attorney wishes to call Buddy, one of Rob¶s friends, to testify about Laura¶s behavior on the night of the alleged rape. Buddy will testify that he went out to dinner with Rob and Laura that night and that Laura was ³very forward´ with Rob²she spontaneously kissed him at least three times and as they were getting up to leave she grabbed his buttocks and squeezed them. The prosecution objects to Buddy¶s testimony. How is the judge likely to rule?

A. Sally¶s testimony is barred by Rule 404, and Buddy¶s testimony is barred by Rule 412. B. Sally¶s testimony is admissible, but Buddy¶s testimony is barred by Rule 412. C. Sally¶s testimony is admissible, but Buddy¶s testimony is irrelevant. D. Both Sally and Buddy¶s testimony are admissible.

************************** Next Question****************************

Chapter 36 - Question# 1 - Steve Austin, a gold-medal sprinter in the 2008 Olympic Games, is now on trial for illegally possessing performanceenhancing steroids in 2007. The prosecution's star witness is Austin's trainer, Dr. Wells, who testifies that he sold Austin thousands of dollars worth of Detrimin, an illegal steroid, in 2007. Austin took the stand in his own defense and admitted that he bought and used the Detrimin but claimed that he did not know what it was; that he trusted Dr. Wells and that Dr. Wells told him Detrimin was a nutritional supplement. In rebuttal, the prosecution calls Oscar Goldman, an old friend of Austin's who testifies that while the two men were training for the 2004 Olympics together, Austin showed Goldman a pill bottle marked 'Detrimin' and said: 'These are some steroids I've started using. You should try some; they've really improved my training regimen.' Austin has never been charged nor convicted of using steroids in the year 2004. Goldman's testimony is: A. Inadmissible hearsay. B. Not barred under the hearsay rule but still inadmissible for any purpose. C. Admissible to prove that Austin had knowledge that he was taking steroids in 2007. D. Admissible to prove Austin has a propensity to use steroids when training. ************************** Next Question****************************

Chapter 36 - Question# 2 - Marcia Brady is on trial for killing her stepfather Michael. In its case-in-chief, the prosecutor introduces a properly authenticated letter from Jan, Marcia's sister. The letter was found in Marcia's possession when she was arrested, and it is dated the day before the murder. In the letter, Jan wrote that Michael had sexually assaulted her numerous times over the past few weeks. The defense objects to this document as hearsay and inadmissible character evidence. The court should rule that the letter is: A. B. C. D. Admissible for any purpose. Admissible to prove Marcia's motive for committing the crime. Admissible to prove that Michael actually did sexually assault Jan. Inadmissible for any purpose.

************************** Next Question****************************

Chapter 39 - Question# 1 - Cynthia is a witness in a kidnapping trial. The prosecutor wants to prove that John, the victim, left his house on at around 8:00 AM on December 19th. The prosecutor call Cynthia to the stand to testify about the morning of December 19, but Cynthia claims the Fifth Amendment and refuses to answer any questions. Which of the following pieces of evidence does Rule 801(d)(1) now allow the prosecutor to introduce? A. Bill's testimony that, 'After John disappeared, Cynthia told me that she looked out our window at 8:00 a.m. on December 19 and saw John leaving his house.' B. The police report noting that, during questioning after John's disappearance, Cynthia reported that she saw John leaving his house at 8:00 a.m. on December 19. C. The police report, but only if Cynthia signed her statement under penalty of perjury. D. Cynthia's testimony to a grand jury, under oath subject to the penalty of perjury, that she saw John leave his house at 8:00 a.m. on December 19. E. None of these. ************************** Next Question****************************

Chapter 39 - Question# 2 - Same fact pattern as above. Assume that the prosecutor calls Cynthia to testify about the morning of December 19. Cynthia does not claim the Fifth Amendment or any other privilege, but she answers ³I don¶t recall´ to every question the prosecutor asks about December 19, John, or any other subject. Which of the following pieces of evidence does Rule 801(d)(1) now allow the prosecutor to introduce? A. Bill's testimony that, 'After John disappeared, Cynthia told me that she looked out our window at 8:00 a.m. on December 19 and saw John leaving his house.' B. The police report noting that, during questioning after John's disappearance, Cynthia reported that she saw John leaving his house at 8:00 a.m. on December 19. C. Cynthia's testimony to a grand jury, under oath subject to the penalty of perjury, that she saw John leave his house at 8:00 a.m. on December 19. D. Cynthia's grand jury testimony, but only if she acknowledges in the courtroom that she remembers appearing before the grand jury. E. None of these.

************************** Next Question**************************** Chapter 39 - Question# 3 - Blair is suing Tootie for battery, alleging that Tootie pushed her down the stairs of the dormitory at their boarding school, causing Blair to break her arm. Before trial, Blair deposes Mrs. Garrett, the headmistress of the school, who was present in the dormitory at the time of Blair's fall. Garrett states in her deposition that the

she heard Blair and Tootie yelling at each other upstairs, and then heard Tootie say: 'I'll make you pay!' and then saw Blair come tumbling down the stairs. At trial, Tootie calls Garrett to the stand. Garrett testifies that she never heard Tootie's voice at all before the accident, and she in fact had no idea that Tootie was present at the time of Blair's fall. On cross-examination, Blair's attorney asks Garrett whether she said in her deposition that she had heard Tootie's voice. Garrett admits that she said that in the deposition, but states that she was mistaken. Blair's attorney then seeks to admit that portion of the deposition into evidence. Tootie's attorney objects. The evidence is: A. Admissible for any purpose. B. Admissible, but with a limiting instruction stating it can only be used to impeach Garrett, not for the truth of the matter asserted. C. Admissible if Blair's attorney has offered some evidence that Garrett had a motive to fabricate her testimony, and then with a limiting instruction stating it can only be used to impeach Garrett, not for the truth of the matter asserted. D. Inadmissible for any purpose.

************************** Next Question****************************

Chapter 40 - Question# 1 - At 3:15 p.m. on December 12th, Delores telephones Gloria to set up a tennis date. Gloria's husband Frank answers the phone and Delores asks to speak to Gloria. Frank says: 'Gloria's not in the apartment right now, I'll have to ask her to call you back.' At precisely the same time across town, a woman driving a black Mercedes runs a red light and strikes a young child, killing him. The Mercedes drives off without stopping. An eyewitness is able to read the first three numbers on the license plate, and the police track the car to Gloria, who owns a black Mercedes with the same three numbers on the license plate. Gloria is prosecuted for vehicular manslaughter and leaving the scene of a fatal accident. At trial, Gloria's attorney calls Frank to the stand. Frank testifies: 'I was home all day on December 12th, and Gloria was with me there the whole time.' On cross-examination, the prosecutor asks Frank whether he ever told Delores that Gloria was not home. Frank denies making the statement. The prosecutor then calls Delores, who will testify about Frank's statement to her on the phone. The defense objects to this proposed testimony. What is the proper ruling? A. Delores cannot testify about Frank's statement, because her testimony would be extrinsic evidence about a collateral matter. B. Delores cannot testify about Frank's statement, because she did not see him make the statement in person and so could not evaluate his credibility.

C. Delores can testify about Frank's statement, but her testimony is admissible only to impeach Frank's earlier testimony, not for the truth of the matter asserted. D. Delores can testify about Frank's statement, and the testimony is admissible both to impeach Frank and for the truth of the matter asserted. ************************** Next Question****************************

Chapter 40 - Question# 2 - George bought an 'original Ansel Adams photograph' from Wendy on eBay. As part of the deal Wendy wrote: 'If you are not satisfied with the item, just send it back and ask for your money back within 30 days of delivery and I'll give you a full refund.' Two weeks after George received the photograph, he sent Wendy an e-mail that said: 'I am so angry with you I feel like throwing things against a wall!!! I did some tests on this photo last week and I discovered it is just a cheap reproduction of an Ansel Adams picture! I am demanding my money back and I am going to destroy your seller rating!' Later that day, George returned the photo. Wendy never refunded George's money, and he sued her for breach of contract. At trial, George offered his e-mail as evidence that (1) the Ansel Adams photograph is not an original; and (2) he demanded his money back within the two-week time frame. Wendy objects to the e-mail as hearsay. What is the proper ruling? A. The entire e-mail is inadmissible. B. The section of the e-mail in which George states that the photo is a cheap reproduction is admissible, but the rest of the e-mail is not. C. The section of the e-mail in which George demands his money back is admissible, but the rest of the e-mail is not. D. Both the statement that the photo was a cheap reproduction and the statement in which George demands his money back are admissible. ************************** Next Question****************************

Chapter 43 - Question# 1 - Fred is on trial for robbing Sam's Electronics. The prosecution alleges that Fred broke into the warehouse behind the store and stole over $10,000 worth of TV's and stereos. To prove the number and value of the items that were stolen, the prosecution calls Gloria, who works as a stock clerk in the warehouse. She testifies that when she came to work the day after the robbery, she noticed that large amounts of inventory were missing. Gloria spent the entire day cataloguing the missing items, and at the end of the day put together a list which included twelve plasma televisions, thirty-two stereo systems, and eighteen digital video recorders. She gave the list to her manager.

At trial, Gloria testifies that (1) she is unable to remember exactly what was stolen from the store, and (2) seeing the list would help to refresh her recollection. The prosecutor can: A. Show her the list and then take it away and have her testify from her 'refreshed recollection.' B. Have her read the list to the jury. C. Admit the list into evidence. D. (A) and (B), but not (C).

************************** Next Question****************************

Chapter 43 - Question# 2 - After the New York Knicks won the NBA championship, a riot broke out in the streets of New York surrounding Madison Square Garden. The riot was observed by Felix Unger, who was watching the tumult through the window of his sixth-floor apartment. A few hours later, knowing that the police might later need assistance in investigating what happened, Unger wrote down notes about what he had seen, e.g. ³Man in green coat smashed out window of sporting goods store,´ and ³Man with red hat set the black Honda on fire.´ Unger delivered the notes to the police department the next day and indicated his willingness to testify if needed. The police identify Vinnie as one of the rioters, and he is charged with setting fire to a black Honda Accord beneath Unger¶s window. At trial, the prosecutor presents evidence that Vinnie was present at the time of the riot and was the only one on the street wearing a red hat. Which of the following statements is true about Unger¶s notes?

A. If Unger testifies, but is unable to remember what he saw that night and states that the notes he wrote would help him remember, he can look at the notes to refresh his recollection. B. If Unger testifies, but is unable to remember what he saw that night, and he states that the notes were accurate when he wrote them and that he wrote them when his memory was fresh, he can read the notes to the jury. C. If Unger testifies for the defense that he never saw anyone with a red hat on the street, the prosecution can admit Unger's notes to impeach his testimony. D. All of the above.

************************** Next Question**************************** Chapter 43 - Question# 3 - Lucy and Ricky are getting divorced, and they are fighting for custody of their twelve-year-old son. Ricky alleges that Lucy once left their son alone in the car for four hours while she was inside a casino gambling and drinking. To prove the allegation, Ricky¶s attorney calls the child (named ³Little Ricky´) to the stand, and

Little Ricky testifies about the incident. The attorney then shows Little Ricky his diary and asks if he wrote about the incident, if he did so when his memory about the incident was fresh, and if he accurately described the incident. Little Ricky agrees to all this. The attorney then asks the son to read from the diary.

A. Inadmissible under Rule 404(a) because it is a prior bad act which is irrelevant to credibility. B. Inadmissible under the hearsay rule because it is an out-of-court statement with no exception that applies. C. Inadmissible because Little Ricky is too young to have capacity to testify. D. Admissible.

************************** Next Question**************************** Chapter 44 - Question# 1 - Phil was a stamp collector who owned over 10,000 rare stamps, which he kept stored in his basement. Three years ago he decided to insure his collection, and he convinced his friend Adelaide to help him catalogue all of his stamps. Adelaide sat at a desk in the basement with the insurance forms while Phil went through each stamp and described its picture, denomination, and condition. Adelaide wrote down exactly what Phil said for each stamp on the insurance forms. When they were done, Adelaide gave Phil the forms and he sent them off to the insurance company. Phil never checked Adelaide¶s work, and Adelaide never looked at a single stamp. Last month Phil¶s basement flooded and all of his stamps disintegrated. The insurance company is refusing to pay his claim because it states there is no real proof of how many stamps Phil had and what kind of stamps they were; there were so many stamps that neither Phil nor Adelaide can remember any details. Phil sues the insurance company, and at trial he would like to admit the insurance forms that Adelaide filled out. But the insurance company states that the forms are merely hearsay and are not admissible in court. How should the judge rule?

A. B. C. D.

The forms should be admitted into evidence. Phil can read the forms to the jury. Adelaide can read the forms to the jury. The forms are inadmissible by any method.

************************** Next Question****************************

Chapter 45 - Question# 1 - An oil well exploded in Western Colorado, killing three workers. The well was owned by Denver-Carrington, a large

multi-national oil company. The families of the workers are suing Denver-Carrington, claiming the company was negligent in maintaining the oil well. In its defense, Denver-Carrington offers two documents. The first is a monthly maintenance report filled out by the company's regional safety inspector, indicating that the oil well was inspected and found to be in good working condition every month from the day it was built until the day it exploded. The second is an e-mail, written by the same inspector to the CEO of the company, which was written a few hours after the incident and provides a physical description of the damage to the pump and the surrounding area. The plaintiffs object to both documents as hearsay. A. B. C. D. The judge should:

Preclude both documents. Admit the monthly maintenance report but not the e-mail. Admit the e-mail but not the monthly maintenance report. Admit both documents.

************************** Next Question**************************** Chapter 45 - Question# 2 - Sunshine Watersports sued its Orlando franchise for underreporting its annual income. Under the franchise agreement, Sunshine Orlando must pay 20% of its gross income to Sunshine Watersports. The parent company claims that Sunshine Orlando earned $15 million last year, but only reported earnings of $5 million to the parent company. At trial, Sunshine Orlando calls its chief accountant to the stand. Sunshine Orlando¶s attorney shows him a pile of documents, and the accountant identifies them as ³daily tallies,´ which are kept by the directors at each of Sunshine Orlando¶s five different locations. The accountant explains that at the end of every shift the location directors receive reports from each of their unit managers; those reports indicate how much money the unit made during the shift. For example, the jetski rental unit may report income of $3,000; the parasailing unit may report an income of $1,500, and so on. At the end of each day, each of the five location directors uses these unit reports to calculate the total income for that location during that day; the director then sends the daily tally to the accounting office. Sunshine Orlando moves to enter the daily tallies into evidence, but Sunshine Watersports objects. What is the proper ruling?

A. The daily tallies are admissible as business records. B. The daily tallies are recorded recollections, and can be read to the jury, but not admitted into evidence. C. The daily tallies are admissible as present sense impressions. D. The daily tallies are inadmissible because they are self-serving.

************************** Next Question**************************** Chapter 45 - Question# 3 - Barney is suing Dr. Spencer for malpractice. Barney claims that he came to Dr. Spencer two years ago complaining of an itchy sore on his arm that would not heal, and that Dr. Spencer told him it was only a mosquito bite. Barney later learned that he had skin cancer and was forced to undergo months of expensive and painful treatment because the cancer had not been detected early enough. At trial, Barney calls Nurse Givens to the stand. Givens worked for Dr. Spencer two years ago and was present at Barney¶s appointment. Givens says she does not remember what Barney told Dr. Spencer, but that she took accurate notes at the end of every appointment about what the patients complained about and what Dr. Spencer told them in return. Her notes for this appointment state: ³Patient Barney complained of a permanent sore that he kept scratching. Dr. S. said it was some kind of bug bite and prescribed an ointment.´ Barney would like to admit these notes to the greatest extent possible. Which of the following can he achieve with the notes? A. Admit them in evidence in their entirety. B. Ask the nurse to read the notes to the jury, but not admit any part of them. C. Ask the nurse to read the notes to the jury, and admit the first sentence of the notes as evidence. D. Nothing; the notes cannot be read to the jury or admitted into evidence.

************************** Next Question****************************

Chapter 45 - Question# 4 - Plaza Hotel sued Plaza House Hotel for infringement of its trade name. To establish a likelihood of name confusion, Plaintiff Plaza Hotel calls a records custodian to lay a foundation for a series of memoranda which the company had asked its employees to prepare at the end of each day listing instances during the day in which telephone callers, cab drivers, customers, and others had confused the two names. Plaintiff began asking for these memoranda after it filed the lawsuit against the defendant. The memoranda should be: A. Excluded, because than probative. B. Excluded, because C. Admitted, because activity. D. Admitted, because they are more unfairly prejudicial and confusing they are hearsay not within any exception. they are records of regularly conducted business they are past recollection recorded.

************************** Next Question****************************

Chapter 46 - Question# 1 - Samuel Drexley, an inmate in the Cansford Maximum Security Prison, is suing the Department of Corrections, claiming that the conditions in Cansford are inhumane. As part of his case, he seeks to introduce a report made by Amnesty International, a private, non-profit organization that conducted an investigation of Cansford last year. Some of the findings in the report include: (i) inmates at Cansford are only given one hour of exercise a week; (ii) the food served is ³generally unfit for consumption´; and (iii) the ³overall conditions at the prison are severely detrimental to the inmates¶ mental and physical health.´ How should the judge rule?

A. The entire report is admissible under 803(8). B. Facts such as (i) are admissible under 803(8), but opinions and conclusions such as (ii) and (iii) are not, and need to be redacted from the report. C. Facts and opinions such as (i) and (ii) are admissible under 803(8), but conclusions such as (iii) are not, and need to be redacted from the report. D. The entire report is inadmissible under 803(8).

************************** Next Question****************************

Chapter 46 - Question# 2 - A lion escaped from its enclosure in the San Diego Zoo and injured seven people in the vicinity. Pursuant to its statutory authority, the California Department of Parks (³CDP´) sent an investigator to determine the cause of the incident. The investigator examined the enclosure, spoke to over a dozen individuals who witnessed the escape, and interviewed a number of zookeepers who helped care for the lion. The investigator submitted a report to CDP that included the following three statements: I. A four-foot hole had been ripped in the fence surrounding the enclosure. II. It appears the lion tore open the hole, which was possible because of a weakness in the fence due to improper maintenance by the San Diego Zoo personnel. III. One of the witnesses to the escape told me that he saw the lion scratching at the weak spot on the fence for ten minutes before the lion escaped. One of the injured parties sued the San Diego Zoo for negligence and wishes to enter the CDP report into evidence. Which, if any, of the statements in the report is admissible?

A. B. C. D.

I only. I and II only. I, II, and III. None of them.

************************** Next Question****************************

Chapter 47 - Question# 1 - The Internal Revenue Service prosecuted Tony Trump for failing to pay sufficient tax on the sale of his Happy Donut stock. To show the original cost of the stock, Trump introduced a letter written by his father on April 6, 1980. The letter read: ³Dear Tony, I am transferring all of my Happy Donut stock to you. You can consider this a gift equal to the current value of the stock. I checked the stock market ticker this morning and that value is $1.24 per share. It may never be worth more than this, but it is all I have. Your loving father, Fred.´ Assume the trial is occurring today. What is the best argument for admitting the letter to prove the value of the Happy Donut stock on April 6, 1980? A. The B. The C. The D. The market letter is letter is letter is letter is report. admissible admissible admissible admissible as as as as the father's state of mind. a business record. a present sense impression. an ancient document, which refers to a

************************** Next Question**************************** Chapter 49 - Question# 1 - Darla was running the Las Vegas Marathon ("LVM") when a car turned onto the marathon course and struck her, breaking both of her legs. Darla sued the car driver for negligence and LVM (the marathon sponsors), for failing to block off the cross-streets on the marathon course. When Darla's attorney deposed the car driver, the driver conceded that there were numerous pylons blocking off the marathon course, but that he thought the marathon was over and was in a hurry to get where he was going, so he turned onto the marathon course anyway. Darla settled with the car driver and called the driver to the stand during the trial against LVM. The car driver testified that he never saw any pylons marking off the marathon course. During LVM's defense case, LVM offered the car driver's deposition testimony in its entirety to prove the fact that there were pylons in place. The prior statement is: A. Admissible under 804(b)(1). B. Inadmissible under 804(b)(1) because statements in a deposition do not count as "testimony." C. Inadmissible under 804(b)(1) because Darla had no chance to crossexamine the car driver during the deposition.

D. Inadmissible under 804(b)(1) because the driver testified at the trial. ************************** Next Question****************************

Chapter 49 - Question# 2 - Same fact pattern as in the previous question. Assume that the judge rules that the car driver¶s prior statements are inadmissible under 804(b)(1). Can LVM invoke any other hearsay exception to admit the driver¶s prior statements for the truth of the matter asserted? A. No, they can only be used to impeach the car driver. B. Yes, they are admissible under 801(d)(1)(A) as a prior inconsistent statement. C. Yes, they are admissible under 803(5) as a recorded recollection. D. They are admissible under both 801(d)(1)(A) and 803(5). ************************** Next Question****************************

Chapter 50 - Question# 1 - Ritchie Cunningham was walking to school when he was suddenly struck by a motorcycle that had ridden up on the sidewalk. The motorcycle then speeds off. Ritchie hits his head on the cement and blood starts coming out of his ears and mouth. About ten minutes later, Ritchie's friend Warren 'Potsie' Weber is walking by and sees Ritchie bleeding on the sidewalk. He runs over to Ritchie, who sits up with the last of his strength and grabs Potsie, saying in a clear and calm voice: 'Potsie, I don't think I'm going to make it. Tell everyone that Arthur Fonzarelli hit me with his motorcycle.' Ritchie then passes out. Potsie is overwhelmed by the tragedy, and starts crying uncontrollably. When the ambulance arrives, he is still sobbing and he screams hysterically at the paramedic: 'Ritchie told me it was Fonzarelli that did this! He said he hit him with a motorcycle!' Ritchie dies on the way to the hospital. Fonzarelli is arrested later that day and is now charged with murder. Charges Potsie does not testify at trial. Instead, the prosecutor calls the paramedic to testify as to what Potsie said at the scene. What is the most likely ruling? A. All of Potsie's statements are inadmissible hearsay. B. Potsie's second statement is admissible to prove that the injury was caused by a motorcycle, but the first sentence (describing who was driving the motorcycle) is not admissible. C. All of Potsie's statements are admissible only if the prosecution can prove that Potsie is unavailable D. All of Potsie's statements are admissible for any purpose.

************************** Next Question****************************

Chapter 50 - Question# 2 - Charles was wheeled into the emergency room on a stretcher. He was bleeding badly from the head and coughing up blood. When a nurse came over to examine him, Charles grabbed her by the sleeve and asked, 'Am I going to survive?' The nurse looked grave and replied: 'We'll do all we can.' Charles became very calm and serious. He looked the nurse straight in the eyes and said: 'I understand what you're saying. I don't have very much time left. You need to know this. My wife hit me with the car in the garage'I was at my tool chest and she gunned the gas; I was pinned up against the back wall.' Charles then lost consciousness. Luckily, Charles survived the ordeal, and the government prosecuted his wife for attempted murder. The prosecutor tries to admit Charles' statement to the nurse. How is the court most likely to rule? A. The entire statement is inadmissible regardless of whether Charles is available. B. Part of the statement is admissible regardless of whether Charles is available. C. The entire statement is inadmissible regardless of whether Charles is available. D. The entire statement is admissible, but only if Charles is unavailable.

************************** Next Question**************************** Chapter 50 - Question# 3 - Sarah¶s body was found at the base of her apartment building; an autopsy confirmed that she died at about 7:00 p.m. on September 14 from injuries sustained in a fall. Sarah¶s window on the twentieth floor was open, and on a table next to her bed was a note written in her handwriting that said: ³I can no longer live with what I have done. I have stolen millions from the family business, and my brother has been falsely accused. Now the entire family faces financial ruin because of me. The only way out is to kill myself.´ The note was dated ³6:45 p.m., September 14.´ Sarah¶s life insurance company refused to pay a benefit for Sarah¶s death because her policy did not cover suicide. The beneficiaries of the policy sued the insurance company, claiming that Sarah fell accidentally from the window. Meanwhile, Sarah¶s brother Greg comes to trial on charges that he embezzled money from the family business. Sarah¶s note is: A. Admissible in Greg¶s trial to prove that Sarah stole money from the family business. B. Admissible in the insurance trial to prove that Sarah jumped, and did not accidentally fall, from the window. C. Admissible in both proceedings. D. Inadmissible in both proceedings.

************************** Next Question**************************** Chapter 51 - Question# 1 - The furnace in Donald¶s basement exploded, destroying the house and severely burning Donald. Donald sued the manufacturer of the furnace, Steamfast Furnace, for unsafe design. Steamfast¶s defense is that the contractor who installed the furnace five years ago did so improperly, placing the furnace in a room that was far too small. Steamfast also claims that Donald knew about the improper installation and assumed the risk of any explosion. The contractor¶s president, Edward, died last year and the company went out of business; neither Edward nor the company is a party to the lawsuit. At trial, Steamfast offers a letter written by Edward to Donald just before the furnace was installed. The letter says: ³I have advised you not to install a Steamfast furnace because your basement is too small to handle that product. The furnace won¶t have adequate space to vent, and the whole thing could explode. We¶re violating municipal code standards by doing this. But it¶s your money and your house, so we¶ll go ahead and install the furnace you¶ve insisted on.´ The letter is properly authenticated, but Donald¶s attorney objects to it as hearsay. The letter is:

A. Inadmissible for any purpose. B. Admissible to show knowledge on the part of Donald, but inadmissible to show that the installation was improper. C. Admissible to show both knowledge on the part of Donald and that the installation was improper. D. Admissible to show knowledge on the part of Donald, and admissible to prove that the installation was improper only if circumstances corroborate the trustworthiness of the letter.

************************** Next Question****************************

Chapter 51 - Question# 2 - Steven is accused of stealing money from Delores, his ex-girlfriend. Delores testifies for the prosecution. In his defense, Steven calls his friend Jill to the stand, who testifies that Delores told her: ³I sure got back at Steven for cheating on me. Once the jury hears the story I made up, they¶ll send him to prison for a long time.´ The prosecution objects to Jill¶s testimony as hearsay. The testimony is:

A. Inadmissible. B. Admissible only to impeach Delores. C. Admissible to impeach Delores and for the truth of the matter asserted.

D. Admissible to impeach Delores, and admissible for the truth of the matter asserted only if the judge determines that the circumstances corroborate the trustworthiness of the statement.

************************** Next Question**************************** Chapter 52 - Question# 1 - Chris is on trial for robbing a jewelry store. Three days before his trial is set to begin, an unknown hit-and-run driver kills the jewelry store security guard who would have been a key eyewitness against Chris. The prosecutor wishes to admit the security guard¶s grand jury testimony against Chris at trial. Is this permissible?

A. Yes, since the grand jury testimony was given under oath at a prior proceeding. B. Yes, but only if the prosecutor can prove by a preponderance of the evidence that Chris was the hit-and-run driver and that he intended to make the guard unavailable. C. Yes, but only if the prosecutor can prove by a preponderance of the evidence that (i) Chris either was the driver or acquiesced in the driver¶s act, and (ii) in doing so, Chris intended to make the guard unavailable for Chris¶ trial. D. No, the security guard¶s former testimony is inadmissible hearsay. ************************** Next Question****************************

Chapter 52 - Question# 2 - Debra sued Renew Homes for breach of contract; she claims that Renew Homes installed used roofing tiles on her house instead of new tiles as specified in the contract. Debra deposed Gabriel, a Renew Homes employee who left the company after work was completed on Debra's home. At the deposition, which Renew Homes's attorney attended, Gabriel stated that his boss at Renew Homes ordered him to rip roofing tiles off a house that was being torn down and deliver those tiles to Debra's house for re-installation. Shortly after this deposition, Renew Homes re-hired Gabriel and gave him a position as a site manager, a promotion from his earlier position. Debra's claim has come to trial, and Gabriel refuses to testify against Renew Homes; he persists in this refusal even when threatened by the court with contempt. Debra now seeks to admit Gabriel's statements from his deposition. These statements are: A. Admissible. B. Inadmissible because Gabriel is not 'unavailable' under the 804(a) definition. C. Inadmissible because Renew Homes did not engage in 'wrongdoing' by rehiring and promoting Gabriel.

D. Admissible only if Debra has evidence that Renew Homes re-hired Gabriel with the intent to deter him from testifying. ************************** Next Question****************************

Chapter 53 - Question# 1 - Eddie Haskell goes on a drinking binge and, in a drunken stupor, runs out of his house and jumps onto the hood of his neighbor's car, slamming it with his foot and denting it badly. The next morning he writes a letter to Ward, the owner of the car, in which he states: 'I am the person who jumped on your car. I am really sorry. If I pay for the damage, will you promise not to press criminal charges against me? Thanks! Eddie Haskell.' Ward hands the letter over to the police, who arrest Haskell later that day. Haskell is now being prosecuted for damaging the car, and the prosecutor seeks to admit the letter. What is the proper ruling? A. The letter B. The letter purpose. C. The letter 408. D. The letter is inadmissible hearsay. is inadmissible hearsay but admissible for a non-hearsay is admissible under the hearsay rule but barred by Rule is admissible for the truth of the matter asserted.

************************** Next Question****************************

Chapter 53 - Question# 2 - Harry Bentley picked up his dry cleaning from Jefferson Cleaners and found that a hole had been burned through the collar of his favorite tweed jacket. He sued Jefferson Cleaners for damage to the jacket. Jefferson's defense was that the hole in the jacket already existed before it was dropped off at the cleaners. Which of the following pieces of evidence is LEAST likely to be admitted? A. The laundry ticket for the jacket in question, properly authenticated, filled out by the Jefferson Cleaners worker who accepted the jacket for cleaning. There is a box on the ticket which states: 'Clothing damaged when received by cleaner?' inside the box someone has written 'Small hole in collar.' The ticket is offered into evidence by Jefferson, and it is introduced by a Jefferson employee who testifies that it is company policy that every worker fill out the form completely and accurately when clothing is dropped off. B. A memo, properly authenticated that was sent from the Vice-President of Jefferson Cleaners to all the workers at the company one week after the jacket was burned. The memo states that 'all tweed jackets should hereby be hand-pressed, since using the automatic press carries a risk of creating holes in the material.' This memo is offered in rebuttal by Bentley after the CEO of Jefferson testified in his own defense that 'Our methods of processing clothing at the time of this incident posed absolutely no risk of damaging clothes. In fact, we have not even seen fit to change our methods since this unfortunate incident arose.'

C. An excerpt from the deposition of Thomas Deenan, a friend of Bentley's. Attorneys for Jefferson Cleaners conducted the deposition, and Bentley's attorneys were present as well. Deenan said in his deposition that he went out to dinner with Bentley a month before Bentley dropped the jacket off at Jefferson's Cleaner's, and during the dinner Deenan noticed a hole in the collar of Bentley's jacket. Sadly, Deenan was hit by a bus and killed a week before the trial. Jefferson Cleaners now wants to admit this portion of the deposition in its case-in-chief. D. Bentley's girlfriend, who will testify that the night after Bentley picked up the jacket, he told his girlfriend over dinner: 'Jefferson Cleaners burned a hole in my favorite tweed jacket.' Assume that Bentley also testifies at the trial and is thus available for cross-examination regarding this statement. This is offered by Bentley in his case-inchief.

************************** Next Question****************************

Chapter 53 - Question# 3 - Aragorn and Sauron signed a contract in which Sauron agreed to sell Aragorn 20 precious rings in exchange for a payment of $100,000. When Aragorn received the rings, he found them to be of inferior quality and he asked for his money back. Sauron refused, and Argorn sued. One month before the trial, Sauron called Aragorn on the phone and said: 'Listen, this has gone way too far; this lawsuit is really damaging my reputation. I'm not admitting to anything, but it's possible I accidentally used an inferior alloy in making the rings. Give me two weeks and I'll make you twenty more rings that I promise you'll be satisfied with. I'll also give you $10,000 back for all the trouble. If I do that, will you drop the lawsuit?' Aragorn replied, 'Would you go to $20,000?' and Sauron hung up the phone. At trial, Aragorn wants to testify to Sauron's statement: 'I'm not admitting to anything, but it's possible I accidentally used an inferior alloy in making the rings.' This evidence is: A. B. C. D. Admissible. Barred by the hearsay rule. Barred by Rule 408. Barred by both the hearsay rule and Rule 408.

************************** Next Question**************************** Chapter 57 - Question# 1 - MaryAnn is suing Ginger, alleging that Ginger stole her black silk dress when they were roommates. At trial, MaryAnn calls Thurston to the stand, who testifies that he saw Ginger wearing the black silk dress in question at a party two weeks after Ginger moved out of MaryAnn's apartment. The trial ends in a mistrial, and the case is re-tried.

At the re-trial, Thurston does not testify. MaryAnn seeks to admit a transcript of Thurston's testimony at the first trial. Ginger objects, and the judge overrules the objection. Ginger then calls Thurston's widow Lovey to the stand. Lovey will testify that she had been married to Thurston for forty years and that in her opinion Thurston was a dishonest man who lied about everything. MaryAnn objects to Lovey's testimony, and the judge overrules the objection. Were either of the judge's rulings correct? A. It dependsâ¼´Thurston's prior testimony is admissible only if Thurston were unavailable. But if the prior testimony comes in, Lovey's live testimony should be admitted. B. Yes, both Thurston's prior testimony is admissible whether or not he is unavailable, and Lovey's live testimony should be admissible. C. No. Thurston's prior testimony should not have been admitted no matter what. But if the judge admits it, the judge should admit Lovey's live testimony. D. No. Thurston's prior testimony is admissible, but Lovey's live testimony should be precluded. ************************** Next Question****************************

Chapter 58 - Question# 1 - Frederick Dawson is a controversial documentary maker, and his latest piece purported to show evidence that the mayor of the city was having an affair with a sixteen year old girl. The documentary was aired on a local television station which is owned by Steve Keaton. The mayor sues both Keaton and Dawson for libel, and Keaton settles the case before trial. At Dawson's trial, the mayor calls Mallory to the stand. Mallory is Steve Keaton's daughter, and she will testify that the night before Steve ran the documentary, he told Mallory: "You know how our local news ratings have been suffering recently? Well, tomorrow we're running this piece by Dawson that should really get people's attention. Dawson tells me it's all fiction, but I'll run whatever it takes to get people to watch," Mallory's testimony is most likely:

A. B. C. D.

Admissible no matter what. Admissible only if Steve is unavailable. Inadmissible under the hearsay rule. Inadmissible under Crawford.

************************** Next Question****************************

Chapter 59 - Question# 1 - Which of these assertions is LEAST likely to be appropriate for judicial notice under Rule 201?

A. B. C. D.

The population of the United States is over 300 million people. German Shephards are the most intelligent breed of dogs. An ostrich is a flightless bird. The movie "The Godfather" has won the Academy Award for best picture.

************************** Next Question****************************

Chapter 59 - Question# 2 - Tina, a tenant in a large apartment building, slipped in the lobby and broke her hip when she fell. It was a rainy day and other tenants had tracked water into the lobby, making the floor slippery. Tina sued Gateway Enterprises, her landlord, claiming that the company should have installed rubber mats to absorb water and reduce the risk of falls. While Tina¶s suit was pending, a group of tenants in the building pooled their money and purchased rubber mats for the lobby. They were concerned about falls and did not want to wait for Gateway Enterprises to act. Tina filed a pretrial motion, seeking permission to introduce evidence of the tenants¶ remedy. Gateway objected under Rule 407, which bars admission of subsequent remedial measures. The trial judge had to decide whether Rule 407 bars evidence of thirdparty repairs, an issue that the court of appeals in his circuit had not addressed. The judge ruled that third-party repairs do not fall within rule 407, so Tina could introduce the evidence. As part of his memorandum opinion, the judge wrote: ³I take judicial notice of the importance of tenant associations in remedying dangerous situations. Our legal system should encourage these self-help organizations.´ Gateway filed a motion for reconsideration, objecting to the judge¶s taking of judicial notice as improper under Rule 201. What is Tina¶s best response to that objection? A. Rule 201(c) gives the judge discretion to take judicial notice. B. The importance of tenant associations is not subject to reasonable dispute. C. Rule 201(f) allows the judge to take judicial notice at any time. D. The importance of tenant associations was not an adjudicative fact. ************************** Next Question****************************

Chapter 59 - Question# 3 - Artie Griffin, the plaintiff in a trademark action involving college football insignia, asked the trial judge to take judicial notice that Ohio State University's football team won the national championship in 2002. Griffin showed the judge a book titled 'The Official College Football Yearbook, 2002' to document this fact. The defendants offered no response to this request.

The judge: A. Must take judicial notice if the Yearbook's accuracy cannot reasonably be questioned. B. May take judicial notice if the Yearbook's accuracy cannot reasonably be questioned. C. Must take judicial notice if the Yearbook qualifies as a 'learned treatise.' D. May take judicial notice if the Yearbook qualifies as a 'learned treatise.' ************************** Next Question****************************

Chapter 59 - Question# 4 - Betsy Hisser won a verdict in a sex discrimination suit against her former employer, the Prince & Spaulding law firm. Hisser's verdict rested on Title VII of the Civil Rights Act of 1964, which applies only to establishments that employ 15 or more people. At trial, Hisser offered no evidence about the number of people working for Prince & Spaulding. Prince & Spaulding noted this lack of proof as one issue response, Hisser urged the court to take judicial notice Spaulding employs hundreds of workers. She referred the Martindale-Hubbell, a directory of the legal profession, than 400 lawyers employed at Prince & Spaulding. The court: A. Will not take judicial notice that Prince & Spaulding employs at least 15 workers, because it is too late to take judicial notice on appeal. B. Will take judicial notice that Prince & Spaulding employs at least 15 workers only if Hisser shows that the probative value of that evidence substantially outweighs any unfair prejudice to the firm. C. Will take judicial notice that Prince & Spaulding employs at least 15 workers if the court concludes that Martindale-Hubbell is a source whose accuracy cannot be questioned. D. Will not take judicial notice that Prince & Spaulding employs at least 15 workers, because this is a 'legislative fact' related to a federal statute. on appeal. In that Prince & court to which lists more

************************** Next Question**************************** Chapter 60 - Question# 1 - Charles Murphy died from an overdose of sleeping pills. The insurance company claimed that Charles committed suicide, and refused to make payment on his life insurance policy. Charles¶s wife argued that the overdose was accidental, and sued the insurance company for non-payment.

At trial, the insurance company called Dr. Mears, a clinical psychiatrist who was a close friend of Charles. Dr. Mears never examined Charles as a patient, but she did have lunch with Charles two days before he died. The insurance company did not attempt to qualify Dr. Mears as an expert. Mears testified that on the day she saw Charles, he ³seemed very depressed; in fact, it appeared to me that he displayed all the symptoms of bipolar affective disorder.´ Charles¶s wife objects to this testimony. How should the court rule?

A. None of the testimony is admissible, because Mears was not qualified as an expert. B. None of the testimony is admissible, because it has very little probative value and a high risk of unfair prejudice. C. The statement ³He seemed very depressed´ is admissible, but the rest of the statement is not. D. The entire testimony is admissible, because Mears has the technical and scientific expertise to make such a diagnosis. ************************** Next Question****************************

Chapter 60 - Question# 2 - Doris is a prosecution witness in a bank robbery/murder trial. She testifies that she saw the defendant enter the bank and yell for everyone to get down on the floor. She then testifies that as she was lying on the floor with her head down, she ³heard a noise that sounded like a gunshot.´ When she looked up, she saw the security guard lying on the ground nearby bleeding from the chest. The defense attorney objects to this evidence. Should the judge allow Doris to testify that she heard ³a noise that sounded like a gunshot?´

A. Yes. B. Probably, but the judge may require evidence that Doris has heard a gunshot in her life on some prior occasion. C. Yes, but only if Doris first qualifies as an expert. D. No, because Doris is not certain that the noise was a gunshot.

************************** Next Question**************************** Chapter 63 - Question# 1 - A group of veterans from the Korean War are suing the United States government for exposure to Code Blue, a chemical defoliant used by the United States Army during that conflict. The veterans claim that their exposure to Code Blue caused them to develop lung cancer. The United States concedes that the veterans were exposed to Code Blue, but argues that there the chemical was perfectly safe and could not possibly have caused the injury.

The veterans intend to call Dr. Benjamin Pierce, a world-famous physician and author of many scholarly articles on cancer and its causes. Dr. Pierce has devised a new laboratory test for studying chemicals to determine whether or not they cause cancer. This test has only been accepted by a handful of researchers around the country, but the judge is convinced that it is reliable, since it has undergone peer review and has very low error rates. Dr. Pierce has not personally tested Code Blue, but he has seen the results of various tests performed by lab technicians who applied his new technique. He will testify that it is standard for physicians and epidemiologists to rely on the work of trained lab technicians when conducting studies and reaching conclusions. Based on the results of the tests that he has seen, he is willing to testify that Code Blue can in fact cause lung cancer. What is the proper ruling governing Pierce's proposed testimony? A. His testimony should be barred entirely because the technique he is relying upon has not been accepted by most of scientists practicing in the field. B. His testimony should be barred entirely because he bases his conclusions on hearsay data. C. His testimony about his conclusions should be allowed, but he cannot testify about the underlying data on direct unless the court determines that the probative value in assisting the jury to evaluate his opinion substantially outweighs the prejudicial effect of the hearsay data. D. His testimony about his conclusions AND the description of the underlying data should be allowed on direct examination. ************************** Next Question****************************

Chapter 64 - Question# 1 - Dr. Pierce is an expert who testified on direct that in his opinion the defendant's defoliant, known as "Code Blue," caused the cancer which the plaintiff was suffering from. On cross-examination, the defense attorney asks Dr. Pierce if he is familiar with A Study of the Causes and Treatments of Cancer,' a medical textbook written by Charles Winchester. Pierce says he has and he agrees that Winchester's book is well-respected in the medical field. The defense attorney then directs Pierce's attention to a footnote in Winchester's book which states: 'It is well-known that most defoliants, such as Code Blue and Undertaker, pose absolutely no danger of cancer due to the inert molecular structure of their ingredients.' The plaintiffs object to any reference to the Winchester book. What is the proper ruling? A. The passage in the textbook is inadmissible for any purpose. B. The passage in the textbook may be used to impeach the witness, but not considered for the truth of the matter asserted. C. The passage in the textbook may be read to the jury, and considered for the truth of the matter asserted, but the book itself may not be admitted into evidence.

D. The book may be admitted into evidence and considered for the truth of the matter asserted. ************************** Next Question****************************

Chapter 67 - Question# 1 - Fantasy, Inc. is a multinational corporation that owns hundreds of hotels and luxury resorts around the world. Last December, a tourist at one of Fantasy¶s resorts died when his paraglider malfunctioned, causing him to plummet two hundred feet into the ocean. The tourist¶s family is suing Fantasy for negligence. Mr. Roark is the general counsel for Fantasy, Inc. He immediately flies to the resort in question and interviews Tattoo, the Fantasy employee who runs the paraglider operation. Before the interview begins, Roark explains that Fantasy is being sued and needs to know what happened in order to mount an effective defense. He also explains the conversation is privileged and confidential. Tattoo then admits to Roark that he knew the paraglider in question was faulty because it was over ten years old, and that Tattoo had embezzled all the money earmarked for new paragliders and gambled it away. Roark takes notes on the interview. What is the status of the notes that Roark took?

A. They are protected by attorney-client privilege, and are not discoverable unless the privilege is waived by Fantasy, Inc. B. They are protected by attorney-client privilege, and are not discoverable unless the privilege is waived by Fantasy, Inc. AND Tattoo. C. They are protected by work-product privilege but not attorney-client privilege; however, they are still not discoverable unless the privilege is waived by Fantasy, Inc. and Tattoo. D. They are protected by work-product privilege but not attorney-client privilege, and they are not discoverable unless the privilege is waived by the appropriate parties OR the plaintiffs can show that they have a substantial need for the information and cannot acquire it by other means without undue hardship. ************************** Next Question****************************

************************** End of Questions ****************************

************************** Begin Answer Key **************************** ************************** Begin Answer Key **************************** ************************** Begin Answer Key ****************************

Chapter 2 - Question# 1 - (B) is the correct answer. The plaintiff claims that the defendant used the club to administer the beating; that makes the club a physical object used in the disputed incident. The plaintiff must authenticate the club in some manner, probably by testifying that she recognizes its distinctive features or picked it up after the beating and kept it in a secure place. Assuming that the plaintiff satisfies the authentication rules, which we will discuss later in the course, the judge will admit the club as real evidence.

(A) is incorrect because the club is not a model or reproduction; it is the actual tool used by the defendant. (C) is incorrect because the club itself is not oral testimony of any kind. The plaintiff was an eyewitness to the event, and she may offer oral testimony about the club. But the club itself is a piece of physical evidence.

(D) is incorrect: A stipulation is an agreement between the parties about the truth of a particular fact. Depending upon other evidence in the case, the defendant in this assault case might stipulate that the club was the one used to beat the plaintiff'while claiming, perhaps, that someone else wielded the club. A stipulation like that would eliminate the plaintiff's need to authenticate the club. But the club itself would remain a piece of real evidence shown to the jury; it would not be a stipulation. ************************** Next Answer**************************** Chapter 2 - Question# 2 - (D) is the correct answer. Rather than risk taking real narcotics into the courtroom, prosecutors sometimes use a model to show the jury how the defendant packaged and sold the drugs. Powdered sugar looks a lot like cocaine, so a plastic bag of powdered sugar is demonstrative evidence used to illustrate the contraband.

If the defendant attempted to palm off powdered sugar as cocaine, answer (C) might be correct: This bag of sugar could be the actual goods offered for sale by the defendant. However, in that case, the defendant would not have been charged with possession of narcotics, since he would in fact not have possessed any narcotics. Therefore, the bag of powdered sugar is simply an illustrative prop.

Answers A and B are incorrect because the bag of sugar is not oral testimony of any type. An eyewitness or expert might testify about the bag and what it represents, but the bag itself is demonstrative evidence.

************************** Next Answer**************************** Chapter 2 - Question# 3 - (C) is the correct answer. The pricelists are documents that serve as real evidence; they are the actual pricelists distributed by the defendants. The lists, however, do not contain any direct evidence of the defendants' intent or agreement. Instead, the plaintiff will use the lists to urge the jury to infer that the defendants must have intentionally agreed to advertise the same price for each product. The number of identical prices, plaintiff will argue, is too large to result from coincidence. This is a circumstantial use of the pricelists.

Answers (A), (B), and (D) are wrong because they suggest that the evidence is demonstrative (rather than real) and/or that the pricelists constitute direct evidence of price-fixing. ************************** Next Answer**************************** Chapter 5 - Question# 1 - (C) is the correct answer. A motion to strike is the proper motion when inadmissible evidence has already entered the case. Since the witness has already volunteered information that violates the privilege, defense counsel will move to strike that testimony from the record. The judge may also give the jury a curative instruction, telling them to disregard the answer.

A motion in limine (answer A) is an evidentiary motion filed before trial. An objection (answer B) is an evidentiary challenge asserted at trial before evidence is received. An exception (answer D) is an outdated mechanism used to preserve an objection for appeal. The Federal Rules of Evidence eliminate the need for exceptions.

************************** Next Answer**************************** Chapter 5 - Question# 2 - (B) is the correct answer. The judge must give the requested limiting instruction. Rule 105 requires the trial judge to give a limiting instruction if the affected party requests that action. The rule differs from most Rules of Evidence by denying discretion to the trial judge.

Answer (C) is incorrect because the judge does not have discretion on this issue. (D) is wrong because, again, the judge lacks power to deny the requested instruction. The company's counsel should consider whether the instruction will do more harm than good, but that is not an issue for the judge to weigh. Answer (A) is incorrect, because the company's counsel has not attempted to introduce any evidence; a party makes an offer of proof when an opponent objects to evidence.

************************** Next Answer**************************** Chapter 5 - Question# 3 - (D) is the correct answer. Most evidentiary errors support reversal only if they (1) constitute an abuse of discretion and (2) affect a party's substantial right. Answers (A) and (B) are incorrect because they each offer only part of this formula. Answer (C) is wrong because the trial judge need not rule with specificity; the specificity standard applies to a litigant's objection. ************************** Next Answer**************************** Chapter 6 - Question# 1 - (C) is the correct answer. Under 401, for evidence to be deemed relevant and therefore admissible, it only has to make a 'fact of consequence' 'more or less probable.' Under this low threshold, Henry's multitasking at 24th and High makes it a little more probable that he was multitasking during the accident two minutes later, and tends to show him to be an inattentive and careless driver, both of which are 'facts of consequence.'

Answer (D) is incorrect because relevance. Paula need not show Henry was still multitasking at proffered evidence makes such a be without the evidence.

it implies the wrong standard for that it was 'more likely than not' that the time of the accident; only that the fact slightly more likely than it would

Answer (A) is incorrect because timing is not a necessary factor in determining relevance of evidence. Even though the spill occurred several minutes before the accident, it may be relevant if it helps prove a 'fact of consequence.'

Answer (B) is incorrect because Rule 401 does not require evidence to be the direct cause of an accident for it to be admissible; the evidence needs only to have "any tendency" to make a fact "more or less probable." Multitasking while driving has some tendency to show that Henry was distracted--and therefore negligent--so it is admissible. ************************** Next Answer**************************** Chapter 6 - Question# 2 - (D) is the answer. Although relevance is a very low standard, the evidence must make a fact in consequence a little more or less likely. Very often, whether something is relevant will depend on the law that applies to the case. In this lawsuit, the plaintiffs only need prove that Serenity caused their injuries²anything else about the effect of the drug, or about whether Farnsworth was at fault in manufacturing the drug²is irrelevant. (A) is incorrect because the racial bias of the CEO²however despicable²does nothing to help a finder of fact determine whether or not the drug caused birth defects.

(In fact, the inflammatory nature of this evidence would make it even less likely to be admitted under Rule 403, as we will learn in subsequent chapters). If there were additional facts, such as the fact that Serenity was heavily marketed towards non-white patients, the CEO¶s racial bias could conceivably be relevant, but that was not the case here. (B) is incorrect because whether or not Serenity actually helps treat depression is irrelevant to the question of whether it causes birth defects. And (C) is incorrect because the applicable law is strict liability²whether or not Farnsworth conducted tests would tend to prove negligent or reckless behavior, but Farnsworth¶s degree of fault is inadmissible in this case. Therefore, none of these facts are relevant to the lawsuit. ************************** Next Answer**************************** Chapter 7 - Question# 1 - (C) is correct. Courts frequently admit evidence of flight to suggest that a defendant had a guilty state of mind. Evidence of flight almost always is relevant, and the probative value of the evidence usually outweighs any unfair prejudice. The same analysis should apply to hiding from the police. If anything, the probative value of hiding is higher than that of flight, because there are fewer innocent explanations for hiding (as opposed to travel).

(A) is incorrect because the information is clearly relevant. As long as the evidence tends to make the fact in consequence a little more likely, it is relevant, even though there are alternative explanations for the conduct.

(B) is incorrect because the unfair prejudicial effect of this evidence is relatively small. If there is an innocent explanation for the defendant's conduct, he can explain it to the jury.

(D) is incorrect because the evidence is admissible to prove the defendant actually is guilty, not just to prove the defendant is lying. ************************** Next Answer**************************** Chapter 7 - Question# 2 - (B) is correct. The tire iron has some slight probative value, since it (along with hundreds of other items) might have been the murder weapon. Therefore, (A) is incorrect. But courts are very careful with demonstrative evidence, and in this case the slight probative value of admitting a possible murder weapon is outweighed by the substantial prejudice of having the prosecutor show the jury an item found in the defendant's garage as a "potential murder weapon" when there is no evidence to show that it actually WAS the murder weapon. Thus, Rule 403 will likely bar the evidence, and (D) is incorrect. (C) is incorrect because evidence is no more or less likely to be admissible simply because it is circumstantial.

************************** Next Answer**************************** Chapter 7 - Question# 3 - (B) is the correct answer, since it sets out the legal test for admissibility under Rule 403. The evidence is relevant, but because it concerns her husband, it has a low probative value. It also carries a very high risk of unfair prejudice, so the judge will have to determine whether the unfair prejudice substantially outweighs the probative value.

Answer (A) is incorrect because the evidence does have some probative value, which must be weighed against its potential for unfair prejudice. Answer (C) is incorrect because the judge need not determine that the proffered evidence has no unfair prejudice; only that the level of unfair prejudice does not substantially outweigh the probative value of the evidence. Answer (D) is incorrect because it simply sets out the test for relevance; although this evidence is almost certainly relevant under Rule 401, its probative value is very low, so it may not be admissible under Rule 403.

************************** Next Answer**************************** Chapter 9 - Question# 1 - Answer (A) is correct. Redesigning the ladder and adding a warning both count as remedial measures, but Rule 407 only applies to measures that are taken after an injury or harm allegedly caused by an event. Periladder made the changes after Leon's injuries, so the evidence is barred in his case by Rule 407.

Answers (B) and (D) are incorrect because Periladder's remedial measure was made prior to Carl's injury; thus, in Carl's case the remedial measure was not a subsequent remedial measure, and his Rule 407 objection will be overruled. ************************** Next Answer**************************** Chapter 9 - Question# 2 - Answer (B) is correct. Rule 407 only allows Erin to admit evidence of Periladder¶s subsequent modification to prove feasibility if Periladder made the argument that a safer design was not feasible; since Periladder did not do so, the evidence is precluded. However, courts have held that Rule 407 does not apply to remedial actions by third parties, so HCN's modifications would most likely be admissible for any relevant purpose. ************************** Next Answer**************************** Chapter 9 - Question# 3 - (A) is the correct answer. The evidence is admissible to prove ownership or control, because House Depot controverted ownership and control during their case. (B) is incorrect

because House Depot is entitled to a limiting instruction telling the jury to only consider the evidence for the purposes of proving ownership or control and not to prove liability. (C) is incorrect because a written warning is considered a remedial measure, and (D) is incorrect because the evidence is relevant to prove something other than fault. ************************** Next Answer**************************** Chapter 10 - Question# 1 - (D) is the correct answer. Agreeing to send the correct type of wood is not a subsequent remedial measure, so (A) is incorrect. The statement is an implicit admission that Ingalls had promised to send oak wood, and is thus relevant to prove that he was in breach of the contract, so (B) is incorrect. And although this was an offer to settle, there was no dispute at the time the statement was made; Ingalls never stated that he did not have to send the wood to Baker. Thus, Rule 408 does not apply and (C) is incorrect. ************************** Next Answer**************************** Chapter 10 - Question# 2 - The correct answer is (C). Although the employees might have suspected that Mitchell was anticipating a breach of contract or was unable to honor their contract, Rule 408 does not take effect until a dispute has actually arisen. Because a dispute had not yet arisen, Rule 408 did not apply.

(A) cannot be correct, since Rule 408 treats offers to settle in the absence of a dispute as simply admissions. Answer (B) is incorrect because there is no information in the fact pattern to show that this information impeaches Cameron. Answer (D) is incorrect because the evidence is in fact being offered to prove liability'that is, that Cameron and his employees both had the same interpretation of the contract at the time of the signing, and therefore Cameron is now liable for breach.

Note that, while the Rule 408 objection will fail, the evidence is still subject to other rules of evidence, such as Rule 403. ************************** Next Answer**************************** Chapter 10 - Question# 3 - The answer is (A). Karen and Tom were not in compromise negotiations. In order for negotiations to fall within the ambit of Rule 408, they must be an anticipation of the claim being forfeited. Tom had the ability to take Karen's offer of employment at another company and still pursue his claim against MLG, and therefore Rule 408 does not apply.

Answer (B) is incorrect because the negotiations were in connection with a claim that was disputed. Tom's claim is directly related to his lost

employment, and if Karen had offered him the employment opportunity in exchange for dropping the claim there would be no question about the connection between the negotiations and the claim. Answer (C) is incorrect because Karen's suggestion that company management had participated in age discrimination, while not a singularly decisive statement, can still be used to help prove liability in this case. Answer (D) is incorrect because the objection should be overruled. ************************** Next Answer**************************** Chapter 10 - Question# 4 - (B) is the correct answer. Rule 408 does not apply, because the FTC is a public agency and was exercising its regulatory authority. This is a criminal trial, and the compromise negotiations related to the claim of a public agency (the FTC) that was exercising regulatory authority. Therefore, since the statement being offered is not an offer, acceptance, or promise, Rule 408 does not apply.

If the statement the government was attempting to introduce was an offer, promise, or acceptance, then answer (C) would be correct, even though the compromise negotiations related to the claim of a public agency exercising its regulatory authority. Because the information is an 'other statement,' however, it is admissible.

Answer (A) is incorrect because Rule 408 protects all settlement negotiations, not just the ones concerning the present claim. (D) is incorrect because Rule 408 does not apply if the statement was made to a public agency and is then used in a subsequent criminal case, as happened here. ************************** Next Answer**************************** Chapter 11 - Question# 1 - (C) is the correct answer. The recall letter that Blocko sent to the retailers is a subsequent remedial measure, since it was an attempt to remedy a dangerous situation after an injury had occurred. Therefore, the letter is inadmissible under Rule 407. The letter (and check) sent to the parents, however, is not precluded by any rule. It is not an attempt to cure the defect which allegedly caused the illness, so it is not covered by Rule 407. It is not an offer to settle under Rule 408, since there is no dispute at the time the letter is sent, and Blocko does not dispute liability in the letter. And although children became ill as a result of the product, the check is not intended to pay for their medical treatment, and so is not covered by Rule 409. ************************** Next Answer**************************** Chapter 11 - Question# 2 - (B) is correct. Rule 409 bars any offer or promise to pay medical expenses to treat an injury when offered to prove liability for the injury. Thus, the statement: ³Don¶t you worry about that; McGraw will pay for everything´ in response to David¶s concern about medical expenses is clearly covered by Rule 409. Its only

relevance would be to prove liability, and that purpose is barred by Rule 409. Thus, (C) is incorrect. Rule 409 applies whether or not medical care is actually furnished²it covers all offers and promises to furnish medical care²so (D) is incorrect.

However, Rule 409 (unlike Rule 408) does not cover any statements made before or after the offer or promise to furnish medical care. Thus, the statement ³«especially if one of our machines malfunctions´²which implies that the machine actually did malfunction²is not an offer or promise to furnish medical care and is therefore not covered by Rule 409. Therefore, (A) is incorrect.

************************** Next Answer**************************** Chapter 11 - Question# 3 - (B) is the best answer. (D) is incorrect because the evidence does not show that Dr. Gordon is biased (this exception applies when there is evidence that the insurance company is paying a witness to testify). (C) is incorrect because control is not being contested by Dr. Gordon, and the existence of malpractice insurance would not serve as evidence of that control. Some judges would choose (A) and bar the evidence, since the plain language of the Rule states that insurance evidence is not admissible ³upon the issue whether the person acted negligently or otherwise wrongfully.´ Arguably, this evidence is being offered by Gordon to prove that she was not acting wrongfully²i.e., that she did not alter the chart. However, the policy reasons behind Rule 411 strongly support (B) as an answer²Dr. Gordon is not using the existence of the insurance to prove that she was or was not negligent, but for an entirely different purpose²to rebut a claim that she falsified the chart by proving that she had less of an incentive for doing so. Furthermore, the only unfair prejudice that could come from this evidence would negatively impact Gordon (who is offering the evidence), since a jury may be tempted to increase the damages against Gordon if it knows that an insurance company is paying for it. Thus, most judges would admit this evidence and (B) is the best answer. ************************** Next Answer**************************** Chapter 12 - Question# 1 - (D) is the correct answer. Dutch's confession is very probative, so (A) is incorrect. And Rule 410 only applies if the defendant is engaged in plea discussions--that is, if he displayed an actual subjective expectation that he was negotiating a plea, and that expectation was objectively reasonable under the circumstances. Regardless of what Dutch might have subjectively believed, the objective circumstances here do not come close to being a plea bargaining situation. Friday has no authority to bargain, he never offers any specific deal, and Dutch never offers to plead guilty. Furthermore, Friday read Dutch his Miranda rights, which is another indication that the situation is an interrogation, not a plea discussion. So (B) and (C) are incorrect.

************************** Next Answer**************************** Chapter 12 - Question# 2 - (D) is correct. Rule 410 only applies to pleas of guilty which were later withdrawn, or to pleas of nolo contendere (otherwise known as ³no contest´ pleas). The Rule does not apply to completed pleas, so (A) is incorrect. (B) is incorrect because the evidence is extremely probative²by pleading guilty, Sherman admitted that he committed the act²and there is little to no chance of unfair prejudice. (C) is incorrect because although Rule 410 only deals with evidence of criminal plea bargaining, its ban on such evidence applies to both civil and criminal cases. ************************** Next Answer**************************** Chapter 16 - Question# 1 - The correct answer is (A). Rule 612 only permits the adverse party to inspect and admit evidence that has been used to refresh a witness¶s memory for the purposes of testifying. In this case, the witness testified that he did not use the document to refresh his recollection, so Rule 612 does not apply. Rule 612(2) does expressly permit the review and admission of evidence that has been used to refresh memory prior to testimony, so if Wolfe had testified that he reviewed the investigative file in order to refresh his memory, the file would have to be brought in for review, even if it was never used directly at trial. Since Wolfe testified to exactly the opposite, (B) is incorrect. If evidence has not been used to refresh a witness¶s memory (as in this case), it cannot be admitted into evidence under Rule 612, so answer (C) is incorrect. While answer (D) may present a tempting line of reasoning, there is no requirement that a witness rely on a writing to refresh their memory.

(The details of this question were adapted from United States v. Sheffield, 55 F.3d 341 (8th Cir. 1995))

************************** Next Answer**************************** Chapter 16 - Question# 2 - (C) is the correct answer. Rule 612(2) gives a trial judge discretion in this instance, stating that a writing used to refresh recollection prior to trial will only be subject to the Rule¶s disclosure requirements if the judge believes that it is necessary to do so in the interests of justice. Answer (A) is incorrect, because a criminal defendant does not have such a right; and the judge¶s decision not to order disclosure in this case is not an abuse of discretion. Abuse of discretion is a very high standard² the judge¶s decision would have to be unreasonable²and this decision was not.

Answer (B) is incorrect, because Rule 612 expressly states that the trial judge has the discretion to deny the adverse party the right to inspect and/or admit the writings. Answer (D) is incorrect because it is too broad; Rule 612 also applies to documents used before trial, but states that the production and admission of those documents is within the discretion of the trial judge.

(The details of this question were adapted from United States v. Blas, 947 F.2d 1320 (7th Cir. 1991)) ************************** Next Answer**************************** Chapter 16 - Question# 3 - The correct answer is (D). The term ³writing´ in Rule 612 applies to any document or item that is used to refresh recollection²audiotapes, pictures, physical items all count as ³writings´ for the purposes of Rule 612. Therefore (A) is incorrect. And there is no requirement that the witness¶s testimony be an exactly the same as the information contained in the writing; the purpose of using material under Rule 612 is to refresh the witness¶s memory of a past event, not to ³prove´ that the witness¶s testimony is correct by comparing it to a written document. If anything, the fact that the witness in this case did not directly rely upon material in the audio recording proves that the witness is using the material for the purpose Rule 612 was designed for: refreshing memory as opposed to reciting material. Therefore (B) is incorrect. (C) is incorrect because Rule 612 does in fact allow a witness to testify to exactly what was in the writing. Note that the opposing counsel could, if she wished, admit into evidence and then play for the jury the portion of the tape that was used to refresh the witness¶s recollection.

(The details of this question were adapted from United States v. Rinke, 778 F.2d 581 (10th Cir. 1985))

************************** Next Answer**************************** Chapter 17 - Question# 1 - The correct answer is (D). Rule 607 expressly permits either party to question a witness¶s credibility. (C) is incorrect because a question which attacks a witness¶s perception is an impeaching question. (B) is incorrect for two reasons. First, its statement of law is too broad²there are a number of exceptions to the rule against leading questions on direct. Second, the question asked by the plaintiff in this scenario was not a leading question²it did not suggest a specific answer because the witness was free to answer yes or no. (A) is incorrect because Rule 607 abolished the old common law rule of ³vouching,´ and allows any party to impeach his or her own witness.

************************** Next Answer**************************** Chapter 20 - Question# 1 - (D) is the correct answer. This is a crime of falsity, and therefore it is automatically admissible under Rule 609(a)(2).

Answer (B) would be correct if Rule 609(a)(1) were applicable; however, the relevant crime was a crime of falsity and also was not a felony. Answer (C) sets out the test if this were a felony, not a crime of falsity, and the witness was the defendant. None of these things are true.

Finally, answer (A) would be correct if it were not for the fact that the misdemeanor in question does involve dishonesty. The mens rea requirement in the Florida statute (that one must "knowingly" have written a bad check) makes clear that dishonesty was an element of this crime: therefore (A) is incorrect.

************************** Next Answer**************************** Chapter 20 - Question# 2 - (C) is correct. Under Rule 609(c), if a pardon is based on a finding of innocence, evidence of the conviction is no longer admissible no matter what; if it is not based upon a finding of innocence, the conviction will still be inadmissible unless a subsequent felony has been committed. The question does not say why the pardon was issued, but in this case it does not matter, since Lipscomb has not committed a felony since the pardon. Therefore answer (D) is incorrect.

Because this is a felony involving dishonesty that was committed as an adult and confinement ended less than ten years ago and a pardon was issued, there is a conflict between Rule 609(a)(2) and Rule 609(c). In either case the Rule 403 balancing test is irrelevant, therefore answer (A) is incorrect. In a conflict between 609(a)(2) and 609(c), 609(c) must be taken into account first. The justification behind Rule 609(a)(2) is that crimes involving dishonesty are highly probative of a witness¶s character for truthfulness, but if the court has already determined that the witness was innocent of that crime or is rehabilitated then²at least ostensibly²evidence of the prior crime has little or no probative value. Therefore (B) is incorrect.

************************** Next Answer**************************** Chapter 20 - Question# 3 - (A) is the correct answer. Under Rule 609(d), evidence that the accused committed a crime as a juvenile is never admissible.

Answer (B) sets out the correct test for juvenile witnesses if the witness is not the accused, but Mancero is the accused in this case. (C) is incorrect because Rule 609(e) states that the pendency of an appeal does not render evidence of conviction inadmissible. Answer (D) is also incorrect in this case, because it ignores the fact that this conviction occurred when the accused was a juvenile (a burglary conviction could still be admissible under 609(a)(1), even though it was not a crime of falsity, if it had been an adult conviction).

************************** Next Answer**************************** Chapter 21 - Question# 1 - The correct answer is (D). Under Rule 608(2), a witness¶s propensity for truthfulness cannot be bolstered until it is attacked. Therefore (C) is an incorrect statement of the law. And although Thaddeus¶s sincerity has been attacked when Graham brought out evidence of bias, this does not constitute an attack on Thaddeus¶s character or propensity.

Evidence of bias presents a reason why the witness may choose to be untruthful on this particular occasion, but it does nothing to attack a witness¶s character for truthfulness. Therefore (A) is incorrect.

Erin¶s proposed testimony is opinion testimony which is directly relevant to Thaddeus¶s honesty, so (B) is incorrect.

(Some details from Renda v. King, 347 F.3d 550 (3rd Cir. 2003) were modified to fit this question)

************************** Next Answer**************************** Chapter 21 - Question# 2 - (C) is the correct answer. Rule 608 only permits a character witness to testify as to his or her opinion or the reputation of the fact witness¶s propensity to lie or tell the truth.

Specific instances of conduct are not allowed. incorrect.

Therefore (D) is

(B) is incorrect because evidence of past lies is certainly relevant to whether or not he is lying on the stand, especially given the low standard of relevance. Finally, while Rule 608(a)(2) only allows evidence of a truthful character after the character of the witness for truthfulness has been attacked, there are no similar restrictions on evidence of an untruthful character. Therefore (A) is incorrect. Had Lorriane testified as to her opinion that Devon was a liar, or had she testified that Devon had a reputation in the neighborhood for lying, her testimony would be admissible.

************************** Next Answer**************************** Chapter 21 - Question# 3 - B) is the correct answer. 608(a) applies only to witnesses, and Lindemann was not a witness at the trial. Therefore, he has not placed his credibility at issue in this case, and any testimony about his dishonest character is not relevant.

(A) is a correct statement of Rule 608(a)(2), but Rule 608 only applies to witnesses, so answer (A) is incorrect.

Liz¶s testimony does constitute character evidence, so (C) is incorrect. (D) is a completely backwards statement of the law; Rule 608 allows general opinion or reputation testimony, but not evidence of specific conduct. ************************** Next Answer**************************** Chapter 22 - Question# 1 - (B) is the correct answer. Rule 608 permits referring to specific instances of conduct when cross-examining character witnesses, but Patrick was not a character witness for Fahrmore. Rule 608 only permits cross-examination of character witnesses in order to ³test the expertise´ of the character witness; in this case, Patrick never claimed to know anything about Fahrmore¶s character, so the questions are improper.

(D) accurately states Rule 608¶s prohibition against extrinsic evidence, but merely asking about an incident does not constitute extrinsic evidence, so (D) is incorrect. (A) and (D) also accurately states the law as set out by Rule 608, but Rule 608 does not apply to fact witness such as Patrick, so (A) and (D) are incorrect.

************************** Next Answer**************************** Chapter 22 - Question# 2 - (D) is the correct answer. The prosecutor¶s question meets all of the requirements of Rule 608(b) except that it is not ³probative of truthfulness or untruthfulness.´ Evidence that a witness is generous or charitable do not amount to evidence of truthfulness. Just as testimony insulting the general character of the witness is not permitted, neither is testimony that merely lauds the witness for having qualities other than truthfulness. (A) is incorrect because Fahrmore¶s donations to Amnesty International are not probative of truthfulness. (B) is incorrect because merely asking about these donations does not constitute admitting extrinsic evidence. And (C) is incorrect²this is indeed evidence of a ³specific instance´ of conduct, even though it technically refers to several instances of conduct.

************************** Next Answer**************************** Chapter 22 - Question# 3 - (A) is the correct answer. This is exactly the type of cross-examination permitted by Rule 608(b)(2)²a question about a specific instance of conduct, used to ³test´ the character witness¶ alleged expertise about the fact witness¶ character. (B) is incorrect because falsifying income tax returns is probative of untruthfulness; likewise, (D) is incorrect because Pergure¶s statements about Fahrmore¶s honesty opened the door to any questions about Fahrmore¶s dishonest conduct. (C) is incorrect because the question asks about specific conduct, not criminal convictions²and even if Rule 609 did apply, there is nothing in Rule 609 which prevents the attorney from both asking about the prior conviction on cross-examination as well as introducing extrinsic evidence of that conviction. ************************** Next Answer**************************** Chapter 28 - Question# 1 - (C) is the correct answer. Rule 404 bars character evidence if offered to show propensity, and there are no exceptions for general propensity evidence in contract cases. (A) is incorrect because the defendant is allowed to prove his own propensity in a criminal case (provided the propensity is pertinent in the case) under Rule 404(a)(1). (B) is incorrect, because evidence of a witness's propensity to tell the truth or to lie is admissible under Rule 608. And (D) is incorrect because if character is directly relevant in a case, the evidence is not offered to prove propensity, and so is not barred by Rule 404. ************************** Next Answer**************************** Chapter 28 - Question# 2 - (A) is correct. Neither the facts of the prior assault nor the conviction itself are admissible to show Taylor's propensity to commit violent actions, since they are barred by Rule 404. Therefore (D) is incorrect. Even if Taylor testifies, he cannot be impeached with the prior conviction, since it is a misdemeanor, and it is

not a crime of falsity. Thus, it is not admissible to impeach under Rule 609, and (B) and (C) are incorrect. ************************** Next Answer**************************** Chapter 30 - Question# 1 - (A) is the correct answer. Ordinarily, this type of character evidence would not be admissible in a civil case, because it is only relevant to prove propensity and Rule 404(a) bars proof for that purpose in civil cases. In child custody cases, however, character evidence is directly relevant to prove character as an element-that is, to prove that Michael and Gloria are actually bad parents, not simply that they have a tendency to act in a certain way. (See Chapter 26). Since character is an element in this type of case, the evidence is admissible. And under Rule 405, the parties may offer any kind of evidence (reputation, opinion, or specific acts) to prove an element.

(B) is incorrect because the evidence is admissible to prove the parties' characters (and the evidence is not probative of truthfulness in any event), and (C) is incorrect because the act does not really prove motive. ************************** Next Answer**************************** Chapter 30 - Question# 2 - (C) is correct. The evidence is admissible under Rule 404(b) because it tends to prove Ralph's intent to assault his wife. The evidence is also relevant to prove Ralph's propensity for violence (or at least for making violent threats), but this purpose is barred under Rule 404(a), so (B) is incorrect. This testimony has nothing to do with Ralph's credibility, so it is not admissible to impeach, as (D) proposes.

There is a danger of unfair prejudice, since the jury will naturally make improper assumptions about Ralph's propensity for violence--thus, the defense could argue that the evidence is inadmissible under Rule 403. However, since the statement came only a week before the assault, Ralph disputes his intent, and this was a threat to carry out the very crime Ralph allegedly committed, it has a high probative value. Most judges would find that the evidence passes the Rule 403 test, especially since the judge can give a limiting instruction telling the jurors not to consider the evidence as proof of propensity. Thus, (A) is not the most likely outcome. ************************** Next Answer**************************** Chapter 30 - Question# 3 - (B) is correct. (ii) is a prior incident which is substantially similar to the crime that Duke is now being charged for, and so will probably be admitted in the prosecutor's case-in chief to prove identity under Rule 404(b). (iii) is inadmissible during the prosecutor's case in chief, since its only relevance as to whether the defendant committed the crime is general propensity to commit fraud,

and general propensity evidence is inadmissible under Rule 404(a) unless the defendant opens the door by supporting his general propensity. However, (iii) is admissible if Duke testifies, since it is a crime of falsity and is therefore automatically admissible to impeach under Rule 609. Finally, (i) is not admissible in the case-in-chief, since it has no relevance as to whether Duke committed the crime, and under Rule 609 it is still not admissible after Duke testifies, since it is a misdemeanor which is not a crime of falsity. ************************** Next Answer**************************** Chapter 31 - Question# 1 - (D) is the correct answer. Davidson's testimony is proper habit evidence, since it is a specific, repeated response to a particular sort of stimulus. As long as Davidson can testify that he consistently conducts this check, his testimony is admissible even if he cannot remember this particular incident, so (A) is incorrect. Although Davidson does have an incentive to lie, his possible bias goes to weight, not to admissibility; the plaintiff's attorney is free to ask him about this bias on cross-examination, so (B) is incorrect. And there is no unfair prejudice to this testimony, so (C) is incorrect. ************************** Next Answer**************************** Chapter 33 - Question# 1 - (D) is the correct answer. Although Rule 412 generally bars evidence of prior sexual conduct by the victim, 412(b)(1)(B) creates an exception for prior sexual conduct between the alleged victim and the defendant, if offered (as it is here) to prove consent. Therefore (A) and (B) are incorrect. Laura's conduct towards Rob on the night of the alleged rape tends to prove that she consented to sexual intercourse later in the night, and therefore (C) is incorrect. And although past actions on the part of the defendant offered to prove propensity to commit the crime would generally be barred by Rule 404, Rule 413 allows the prosecutor to admit this evidence for any purpose. Although it is possible the judge will bar this evidence on 403 grounds, she is likely to admit it. Thus, (D) is correct. ************************** Next Answer**************************** Chapter 36 - Question# 1 - (C) is the correct answer. The statement is not being offered to prove the truth of the matter that Austin asserted; i.e., to prove that Detrimin is a steroid or that Austin was using steroids in 2004--the former statement is not contested, and the latter statement is irrelevant, since Austin is not being accused of any wrongdoing in 2004. Instead, the evidence is offered to prove that Austin has knowledge of the fact that Detrimin is a steroid--this is a contested, relevant fact and the fact that Austin stated it is a steroid tends to prove that he knew that fact.

(A) is incorrect because the statement is not offered for a hearsay purpose. (D) is incorrect because the statement is inadmissible both under the hearsay rule and under Rule 404 to prove that Austin used

steroids in 2004 in order to prove his propensity. Finally, (B) is incorrect because the statement does have an admissible purpose. ************************** Next Answer**************************** Chapter 36 - Question# 2 - (B) is the correct answer. Since the letter was found on the defendant's person, the letter is relevant to prove that she had a motive to kill Michael. (A) and (C) are incorrect, since the letter cannot be used to prove the truth of the matter asserted--i.e., that Michael actually did sexually assault Jan--because that purpose would violate the hearsay rule. (D) is incorrect because the letter is relevant for a non-hearsay purpose--whether it is true or not, the fact that Marcia read it would provide her with a motive. And the letter is not barred by Rule 404, since it is not being admitted to prove Michael's propensity to commit sexual assault--indeed, it is not even being offered to prove that Michael did or did not assault Jan. ************************** Next Answer**************************** Chapter 39 - Question# 1 - The correct answer is (E). Cynthia has claimed the Fifth Amendment privilege, refusing to answer any questions, so she is not subject to cross-examination. Rule 801(d)(1) allows a party to introduce a witness's prior statement only when the witness is subject to cross-examination. The prosecutor, therefore, cannot introduce any of the statements listed in responses (A)-(D). ************************** Next Answer**************************** Chapter 39 - Question# 2 - The correct answer is (C). Although Cynthia has answered 'I do not recall' to every question, she is still subject to cross-examination as the courts have interpreted that requirement. Her failure to remember, moreover, is inconsistent with her prior statements. The grand jury testimony meets the final requirements of 801(d)(1)(A) because it was given under oath, subject to the penalty of perjury, and at a proceeding. Rule 801(d)(1)(A) does not require the witness to acknowledge the prior statement, as answer (D) suggests. ************************** Next Answer**************************** Chapter 39 - Question# 3 - (A) is correct. Garrett is a witness on the stand, so Rule 801(d)(1) applies to any of her past statements. Her statement during the deposition is inconsistent with her trial testimony, and the statement was made under oath at a prior proceeding, so it is admissible under 801(d)(1)(a). (B) is incorrect because the statement is admissible for the truth of the matter asserted, not just to impeach, since it fits into an exemption to the hearsay rule. (C) is incorrect because there is no requirement of a motive to fabricate under 801(d)(1)(a) for inconsistent statements. (D) is incorrect because the statement is admissible to impeach and for the truth of the matter asserted. ************************** Next Answer****************************

Chapter 40 - Question# 1 - The correct answer is (D): The testimony is admissible both to impeach Frank and for the truth of the matter he asserted. Frank's statement was a present sense impression. When Delores asked to speak to Gloria, Frank replied 'Gloria's not in the apartment right now.' That statement conveyed his immediate perception that Gloria was absent from the apartment.

Because Frank's statement fits within one of the hearsay exceptions, the prosecutor can use Delores's testimony to show the truth of what Frank said. In other words, the prosecutor can argue that, because Frank told Delores that Gloria was not in the apartment, Gloria was in fact out of the apartment when he spoke.

The prosecutor may also offer Delores's testimony under Rule 613, to impeach Frank's courtroom testimony with his prior inconsistent statement to Delores. Rule 613, however, admits a prior inconsistent statement only for impeachment, not to show the truth of the matter contained in the prior statement. The power of a hearsay exception is that it allows use of a prior inconsistent statement to show the substance of what the speaker said.

Answer (A) is incorrect, because Delores's testimony is not collateral. It relates to a central issue in the trial, whether Gloria has an alibi for the crime.

(B) is not correct because the exception for present sense impressions does not require an in-person statement. The exception depends on the rationale that someone relating a contemporaneous impression'as Frank did in this case'has insufficient time to lie. Gloria's attorney, of course, can note that Delores was speaking to Frank over the phone and may not have been able to detect any deception in his statement.

Answer (C) is incorrect because, as explained above, the exception for present sense impressions allows the prosecutor to admit this testimony to show the truth of what Frank said, that Gloria was not home when Delores called. ************************** Next Answer**************************** Chapter 40 - Question# 2 - The correct answer is (C): Only the part of the e-mail in which George demands his money back is admissible. George is not offering this part of the e-mail to prove the truth of the matter asserted. Instead, the demand has legal significance of its own: Regardless of whether George honestly intended to demand his money back

(which would be the 'truth' of this statement), the statement shows that George complied with Wendy's 30-day notice requirement. As Chapter 35 explains, statements like this are not hearsay; they are admissible without the need for an exception.

Answer (A) is incorrect because, as discussed above, George's demand is admissible.

(B) is incorrect because the section of the e-mail stating that the photo is a cheap reproduction is hearsay that does not fall within any exception. The e-mail is a statement that occurred outside the courtroom, and George offers this part of the e-mail to prove the truth of the matter asserted (that the photo is, in fact, a cheap reproduction).

The statement does not fall within the exception for present sense impressions, because George was relating a fact (the nature of the photo) that he discovered some time before making the statement. He was not reporting his contemporaneous discovery that the photo was fake.

Nor does this statement fall within the exception for excited utterances. George says in the e-mail that he is angry, and he uses a lot of exclamation points, but he discovered the photo's status 'last week.' Discovery of the photo fakery might be an exciting event, but no court would find that George's excitement persisted several days to a week after that discovery. George may still be angry, but he has had sufficient time to formulate and utter lies.

(D) is incorrect for the reasons given above: The statement relating to the photo's status is not admissible. To prove that the photo is a cheap reproduction, George must introduce the type of evidence that originally led to that discovery. He cannot simply rely on his own out-of-court statement. ************************** Next Answer**************************** Chapter 43 - Question# 1 - The correct answer is (A). Gloria says that examining the list would refresh her recollection, so the prosecutor can pursue that path. To refresh recollection under Rule 612, the prosecutor would show Gloria the list, then take the list away and have her testify from her refreshed recollection.

The prosecutor could instead try to lay the foundation to admit the list under Rule 803(5), the hearsay exception for recorded recollections. Under that exception, Gloria would read the list to the jury, as answer (B) suggests. For that exception to apply, however, the witness must testify that she had a good memory of what was missing when she made the list. This is probably true in our case, but we need to have the witness testify to this fact in order to lay the proper foundation. ************************** Next Answer**************************** Chapter 43 - Question# 2 - (D) is correct. Anything can be used to refresh a witness's recollection under Rule 612, so (A) is correct. And since Unger wrote the notes when his memory was fresh, he is available in the courtroom, and he testifies that the notes were accurate when written, he can read the notes to the jury as a recorded recollection under Rule 803(5). Therefore, (B) is also correct. And under Rule 613, a witness can be impeached by any prior statement, so (C) is also correct. ************************** Next Answer**************************** Chapter 43 - Question# 3 - The answer is (B). There was no showing that the child had no current memory of the event²in fact, he testified about it²so Rule 803(5) does not apply. (It is possible, but doubtful, that the attorney could lay a foundation for excited utterance²but no such foundation was laid here). (C) is wrong because a twelve year old has plenty of capacity to testify, and (A) is wrong because Lucy¶s character is directly relevant to a child custody case, so 404(a)¶s propensity bar does not apply. ************************** Next Answer**************************** Chapter 44 - Question# 1 - The correct answer is (C). Each of the forms includes two levels of hearsay: Adelaide¶s written statement (filling out the form) and Phil¶s oral statement (describing each stamp). Phil¶s comments are admissible as present sense impressions, and Adelaide¶s statements are admissible as recorded recollections. The latter exception allows a witness to read the recording into evidence so, after laying the appropriate foundations, Adelaide can read the forms to the jury. ************************** Next Answer**************************** Chapter 45 - Question# 1 - (B) is the correct answer. The monthly maintenance reports are records made by an individual with personal knowledge which are kept in the course of a regularly conducted business activity, and it was the regular practice of that business to keep those records. Thus, they are business records and are admissible under Rule 803(6). The e-mail, however, was not a regularly conducted business activity--it was a response to an extraordinary event. Furthermore, the circumstances might indicate a lack of trustworthiness, since the inspector may have known at that stage that litigation was likely, and thus he may have written the e-mail with something other than a business purpose. Thus, (D) is incorrect.

************************** Next Answer**************************** Chapter 45 - Question# 2 - The correct answer is (A). These are perfectly authenticated business records. The accountant is familiar with the business practices of the business and so can serve as the custodian of this information.

(B) is incorrect because the declarant who made the record must testify as to the recorded recollection. Presumably this is impossible because there were dozens of different unit managers who kept track of this information.

(C) is incorrect because the person who made the document did not have personal knowledge²using present sense impression would require each of the managers to personally testify about their own reports, and require that each of them wrote down the number themselves.

(D) is wrong because it makes no difference whether or not business records are self-serving; they are considered reliable because those who make them are in the routine of doing so, and because the company relies on the records being accurate.

This question illustrates why the business records exception is so useful. These records are both probative and reliable, yet there is no other way for the business to put these internal records in front of the factfinder.

************************** Next Answer**************************** Chapter 45 - Question# 3 - The correct answer is (A). Barney almost certainly can lay a foundation to admit these notes under Rule 803(6), the exception for business records. That exception will admit the record itself, as well as the doctor¶s diagnosis and prescription. Although the latter statements constitute hearsay within hearsay, the business records exception covers both levels of hearsay. That exception does not include Barney¶s description of his symptoms, so that statement represents a separate hearsay-within-hearsay problem. Barney¶s statement, however, was one made for medical diagnosis, so it is admissible under 803(4).

Answers (B) and (C) refer to past recollection recorded, but Barney can go further with the business records exception.

************************** Next Answer**************************** Chapter 45 - Question# 4 - (B) is the correct answer. Even if each employee who made the records testified to lay a foundation under 803(5), the records are inadmissible as past recollection recorded, because 803(5) only allows the witness to read from the record, not to admit the document. So (D) is incorrect. The records do not fall within the business records exception (C) because they were prepared in anticipation of litigation; so they fail the trustworthiness requirement. They might also fail the ³regularly conducted business activity´ requirement, because keeping track of customer confusion may not be part of the hotel¶s regular business. (The records also contain third-party statements, but that probably is not an issue because the hotel is not offering the third-party statements for the truth asserted. On the contrary, they are showing that the third parties were confused. Only the fact of the third party¶s statement or action is relevant.) Therefore, the memoranda are hearsay not falling within any exception (B). The 403 issue (A) is moot, since they are hearsay. But in any event, the probative value of these statements in tradename litigation would be high, so A is incorrect. ************************** Next Answer**************************** Chapter 46 - Question# 1 - The correct answer is (D). Amnesty International is not a public agency, so its reports cannot qualify for admission under the public records exception. ************************** Next Answer**************************** Chapter 46 - Question# 2 - The correct answer is (B). This is a civil lawsuit, the California Department of Parks is a public agency, and the agency was acting pursuant to its statutory power. Thus, the ³factual findings´ from the agency¶s investigation are admissible. The Supreme Court has construed factual findings to include opinions and conclusions, so both statement I (a fact) and statement II (the investigator¶s opinion) are admissible. The public records exception, however, does not include statements by third parties that are simply reproduced in the investigation report; to be admissible, those statements must fall within a separate hearsay exception. Statement III is such a third-party statement and it does not qualify for any other hearsay exception, so it is not admissible. ************************** Next Answer**************************** Chapter 47 - Question# 1 - (D) represents the best argument in this case. The letter qualifies as an ancient document, although the reference to the ticker report is hearsay within that ancient document. Tony can argue that the ticker report would be admissible under 803(17), the hearsay exception for market reports. He may have some trouble laying the foundation for that exception, because he must persuade the judge by

a preponderance of the evidence that his father did rely upon the ticker report rather than some other source. But this is Tony's best argument.

(A) is wrong because the state of mind exception does not include statements of memory or belief used to prove the things remembered or believed. Tony could use that exception to admit this letter to prove that his father believed the stock was worth $1.24 per share, but he can't use that exception to admit the letter to prove that the stock was worth that amount.

(B) is incorrect personal records father held this business records

because the business records exception does not apply to or household accounts. There is no evidence that Tony's stock as part of a business. Other aspects of the exception also are missing here.

(C) is incorrect because Tony¶s father said that he checked the ticker that morning; he was not recording the price directly as he looked at it.

************************** Next Answer**************************** Chapter 49 - Question# 1 - The correct answer is (D). Since the driver testified at trial about this subject matter, he is clearly not 'unavailable,' and no 804 exception applies. If the driver did not appear at trial, his deposition testimony would be admissible under Rule 804(b)(1). Depositions do count as prior testimony, and the opposing party (Darla) had the opportunity to develop the testimony during the deposition. ************************** Next Answer**************************** Chapter 49 - Question# 2 - (B) is correct. The driver's deposition testimony is a prior inconsistent statement, made under oath, at a proceeding. The driver is also subject to cross-examination at the current trial, so the statement is admissible under 801(d)(1).

The statement is not admissible under 803(5) (recorded recollection) because that exception applies only if the declarant/witness has no recollection of the information that was recorded. In this case, the car driver testified as to whether there were pylons, so he obviously has his own recollection of what happened. ************************** Next Answer****************************

Chapter 50 - Question# 1 - (D) is the correct answer. The question involves a "double hearsay" statement. Potsie's statement to the ambulance driver is clearly an excited utterance; thus, the second "leg" of the double hearsay statement is not a problem. But this only gets us to the statement "Ritchie told me..." In order to admit the statement for the truth of what Ritchie asserted (which is the only relevant purpose), Ritchie's statement itself must be admissible under the hearsay rule.

Ritchie's statement qualifies as a dying declaration--it was made under belief of impending death, Ritchie is now dead and thus unavailable, and this is a homicide case. Thus, both "legs" of the hearsay statement are satisfied and (D) is correct. Since the evidence is admissible, (A) is incorrect. Since Ritchie's statements are admissible as a dying delcaration, we need not use the medical diagnosis exception to admit any part of the statement (and at any rate there is no evidence that Ritchie intended for this to be used by a medical professional--the paramedics did not appear until long after the statement was made). Thus, (B) is incorrect. Finally, (C) is incorrect because Potsie's statement is admissible as an excited utterance, and that exception is not dependent upon availability. ************************** Next Answer**************************** Chapter 50 - Question# 2 - (B) is the correct answer. This is somewhat of a trick question'the statement to the nurse is potentially both a dying declaration and a statement made for the purpose of diagnosis and treatment. However, it can never be a dying declaration because this is a criminal, non-homicide case. It is still an 803(4) statement and is thus partially admissible'though any reference to his wife would have to be redacted. Thus, part of the statement--the part in which he states the cause of his injury--is admissible as a statement made for medical diagnosis (an exception which does not rely on unavailability), and the rest of the statement is inadmissible no matter what. ************************** Next Answer**************************** Chapter 50 - Question# 3 - (B) is the best answer. The letter is probably a dying declaration and may also be admissible as a state of mind under 803(3) to prove the cause of her death.

A) is incorrect because because--even if a court concluded that the theft was part of the ³circumstances´ that led to her suicide (which would be a stretch)--Greg¶s trial is a criminal trial which is not a homicide, so the dying declaraton rule does not apply. Therefore, (C) is also incorrect.

See State v. Satterfield, 193 W. Va. 503, 457 S.E.2d 440 (W.Va. 1995) for a good discussion on the admissibility of suicide notes as dying declarations.

************************** Next Answer**************************** Chapter 51 - Question# 1 - (C) is correct. The statement is admissible to prove knowledge regardless of whether a hearsay exception applies, since that is not a hearsay use of the statement. Therefore, (A) is incorrect.

The letter is also admissible for the truth of the matter asserted, because Rule 804(b)(3) applies to the letter. Edward, the author of the note, is dead, so he is not available. And it was against his interest to write the note, because he subjected himself to potential civil and criminal liability (violation of the municipal code). Finally, this is a civil trial, so special corroboration is not necessary. ************************** Next Answer**************************** Chapter 51 - Question# 2 - (B) is correct. The evidence is a prior inconsistent statement, and it is not a collateral matter, so extrinsic evidence is allowed. Therefore (A) is incorrect. However, the statement is not admissible as a prior statement of a witness (i.e., for the truth of the matter) because it was not made under oath at a proceeding. Nor is it admissible as a statement against interest because Delores is available and testified at the trial. Therefore (C) is incorrect. If Delores were unavailable, (D) would be correct, since this is a statement against interest that exculpates the accused and could expose Delores to criminal liability for filing a false report and perjury.

************************** Next Answer**************************** Chapter 52 - Question# 1 - (C) sets out the proper standard²Chris, the opposing party, has to have committed the wrongdoing or has to have acquiesced in it. (A) is incorrect because Chris had no chance to crossexamine the security guard in the grand jury, so 804(b)(1) does not apply. And (B) is incorrect because there is no need for Chris himself to have committed the wrongdoing. ************************** Next Answer**************************** Chapter 52 - Question# 2 - (A) is correct. This is a valid 804(b)(1) use of prior testimony'Renew Homes was present at the deposition and had an opportunity and similar motive to cross-examine Gabriel there.

If Gabriel's prior statements had been made outside the deposition context, (C) would be correct, since Renew Homes did not commit a 'wrongdoing' by re-hiring and promoting an employee. (B) is wrong because refusal to testify does constitute unavailability, and (D) is wrong because even if there were the requisite intent, there was no wrongdoing by Renew Homes. ************************** Next Answer**************************** Chapter 53 - Question# 1 - (D) is the correct ruling. There is no hearsay problem with the letter--it is a statement by a party, offered against that party, so it is admissible as a party-opponent statement under Rule 801(d)(2). Thus, (A) and (B) are incorrect. (C) is incorrect because, although this is an offer to compromise, and although Rule 408 applies in criminal cases, there was no "dispute" at the time that Haskell made the statement--this is simply an admission. ************************** Next Answer**************************** Chapter 53 - Question# 2 - (D) is the correct answer. This problem requires you to review a number of different rules. The evidence in (A) is admissible as a business record to the hearsay rule. It is kept in the ordinary course of business and it is the regular business of Jefferson Cleaners to keep these records, and the records were made by a person with personal knowledge. The evidence in (B) is also admissible-it is not hearsay because it is a statement made by an agent of Jefferson Cleaners within the scope of his employment and it is being offered against Jefferson Cleaners--therefore, it is a party-opponent admission. Although it is a subsequent remedial measure, it is not barred by Rule 407 because the CEO of Jefferson Cleaners denied that any change in policy occurred after the incident; therefore, this is admissible to impeach. The evidence in (C) hearsay, but it is admissible under 804(b)(1)--Deenan's statements were made under oath and subject to crossexamination, and Deenan is now unavailable to testify. The evidence in (D) is also hearsay--it is a statement made by Bentley and offered by Bentley to prove the truth of the matter it asserts. It does not fit into any exception. Therefore it is inadmissible hearsay. ************************** Next Answer**************************** Chapter 53 - Question# 3 - (C) is correct. The evidence is a partyopponent admission, so is not barred by the hearsay rule. However, it is a statement made as part of settlement negotiations after a dispute has arisen, and so it is barred by Rule 408. ************************** Next Answer**************************** Chapter 57 - Question# 1 - (A) is correct. Thurston's prior testimony is admissible under 804(b)(1), but only if the proponent of the testimony can prove that he is unavailable. Once the testimony is admitted, the opposing party should be allowed to impeach Thurston in any way which would have been permissible had he testified live. Rule 608 allows the

opposing party to impeach a witness by opinion evidence to prove his propensity to lie, so this testimony should be permitted. (B) is incorrect because Thurston's prior testimony would not be permitted unless Thurston were proven to be unavailable. ************************** Next Answer**************************** Chapter 58 - Question# 1 - (B) is the correct answer. Steve's statement involves double hearsay. Dawson's statement to Steve is a party-opponent admission, since it is being offered against Dawson. When Steve reported Dawson's statement to Mallory, he was making a statement against interest, since it tended to expose him to civil liability, and thus it is admissible under 804(b)(3) only if Steve is unavailable. If Steve is available, 804(b)(3) does not apply, so (A) is incorrect. Since an exception to the hearsay rule applies, (C) is incorrect. And since this is a civil trial, Crawford does not apply, because the Sixth Amendment only applies in criminal cases, so (D) is incorrect. ************************** Next Answer**************************** Chapter 59 - Question# 1 - (B) is correct. Choices (A), (C), and (D) are all facts which are beyond reasonable dispute and 'capable of accurate and ready determination' from a wide variety of reliable sources. (B) is much more of an opinion than a fact--reasonable experts could disagree as to which breed of dog is the most intelligent. Thus, it is inappropriate for judicial notice. ************************** Next Answer**************************** Chapter 59 - Question# 2 - Tina¶s best response is answer (D), the importance of tenant associations was not an adjudicative fact. In response to this pretrial motion, the judge decided an issue of law: whether Rule 407 applies to repairs made by third parties. When deciding that issue, the judge considered the importance of tenant associations. In this context, that fact is a legislative²rather than adjudicative² fact. In other words, it is a fact that informs a judge¶s resolution of a legal issue.

Rule 201 does not govern the use of legislative facts, so the restrictions in that rule do not apply here. Tina¶s best response to Gateway¶s motion, therefore, is to point out that the premise of the motion is wrong; Rule 201 simply does not apply.

The other three answers are incorrect because they assume applicability of Rule 201, which does not govern judicial notice of the legislative fact in this cases. In addition, none of the factors mentioned in these answers is sufficient on its own to support judicial notice. Although a judge has discretion to take judicial notice (answer A), that discretion has limits; the judge must comply with other portions of the rule. In addition to finding that a fact is not subject to reasonable dispute

(answer B), the judge must also find that it is generally known in the jurisdiction or readily ascertainable from reliable sources. And, although Rule 201(f) allows the judge to take judicial notice at any time (answer C), that notice must comply with other portions of the rule.

************************** Next Answer**************************** Chapter 59 - Question# 3 - (A) is correct. Rule 201(d) provides that judicial notice is mandatory if a party makes that request and supplies the 'necessary information.' Griffin has requested judicial notice, so the only question is whether the Yearbook provides the 'necessary information.' Rule 201(b) allows judicial notice of a fact that is capable of accurate and ready determination from a source 'whose accuracy cannot reasonably be questioned.' Answer A incorporates that standard.

(B) is incorrect because, if Griffin has provided the necessary information by offering documentation in a source whose accuracy cannot reasonably be questioned, then the judge must take judicial notice.

Answers (C) and (D) are incorrect because the Rules of Evidence do not govern preliminary determinations, such as whether to take judicial notice. The source offered by Griffin, therefore, need not satisfy the hearsay rules. (D) is also incorrect by suggesting that judicial notice is discretionary in this case. ************************** Next Answer**************************** Chapter 59 - Question# 4 - (C) is correct. The court will take judicial notice of this fact if it concludes that Martindale-Hubbell is a source whose accuracy cannot reasonably be questioned.

(A) is incorrect because Rule 201(f) allows a court to take judicial notice at any time, even on appeal.

(B) is incorrect because the party requesting judicial notice does not have to meet any special probative value/prejudice standard, even when requesting judicial notice on appeal. The 'reverse 403' standard cited here governs admission under Rule 609 of convictions that are more than ten years old. This standard does not apply to any aspect of judicial notice.

(D) is incorrect because the number of workers at Prince & Spaulding is an adjudicative fact. If this were a legislative fact, Rule 201 would not apply at all, and the court would be free to take judicial notice according to its own discretion. ************************** Next Answer**************************** Chapter 60 - Question# 1 - (C) is the correct answer. Even though she could be qualified as an expert, Mears can still testify as a lay witness, as long as she gives opinions which are helpful to the jury, rationally based on her personal observation, and do not require technical knowledge or specialized process of reasoning. The statement that someone ³seemed very depressed´ is an everyday observation that any lay person could make, so (A) is incorrect. (D) is incorrect because although Mears possesses the necessary knowledge to make such a statement, she was never qualified as an expert in this case. (B) is incorrect because the fact that the decedent appeared depressed a few days before his death has a very high probative value in determining whether he killed himself, and the evidence carries very little risk of unfair prejudice. ************************** Next Answer**************************** Chapter 60 - Question# 2 - (B) is the best answer. Witnesses can testify to opinions to describe what they perceived, so (D) is incorrect. This is a proper lay opinion, since it requires no technical knowledge or specialized process of reasoning, so (C) is incorrect. However, in order to show that the witness knows what a gunshot sounds like (since this is not a sound that everyone is familiar with), a judge may require the attorney to first lay a foundation by demonstrating that the witness has heard a gunshot before this incident. Some judges may not require this, reasoning that hearing a gunshot on television or in the movies is a sufficient foundation, and gunshots appear so frequently in those media that there is no need to lay a foundation. However, a judge who has familiarity with guns and knows how different they sound in real life as opposed to in the movies would likely require the attorney to lay a foundaton. ************************** Next Answer**************************** Chapter 63 - Question# 1 - (C) is correct. The expert testimony is admissible under the Daubert test, since the judge has concluded after considering all of the Daubert factors that the scientific technique is reliable. Since the Frye test has been overruled, the fact that the technique has not been generally accepted is not dispositive. Therefore, (A) is incorrect. (B) is incorrect because he is permitted to base his conclusions on hearsay evidence if that is the standard for experts in the field, as he said that it was. Finally, (D) is incorrect because the underlying data is not automatically admissible; it is only admissible if it passes the test outlines in (C). ************************** Next Answer****************************

Chapter 64 - Question# 1 - (C) is the correct answer. Dr. Winchester's book is admissible under the learned treatise exception to the hearsay rule, which allows text from a learned treatise to be read into evidence and considered for the truth of the matter asserted, but not for the treatise to be admitted into evidence. (See Chapter 47 for details on this exception). A text will be considered a "learned treatise" if a qualified expert such as Dr. Pierce has certified it as such. (A) is incorrect because the learned treatise exception applies, and (B) is incorrect because the exception allows the evidence to be admitted for the truth of the matter asserted, and (D) is incorrect because the exception does not permit the treatise itself to be admitted into evidence. ************************** Next Answer**************************** Chapter 67 - Question# 1 - (A) is the correct answer. The information gathered by the general counsel passes the Upjohn test, since (1) the information was requested by the general counsel so that he could provide legal advice to the corporation; (2) the information concerned matters within the scope of the employees' duties; (3) the information sought was not available to the upper-level management; and (4) the employees knew that they were being questioned in order for the corporation to receive legal advice.

(B) is incorrect because the attorney-client privilege is controlled by the corporation, not by the individual employees who made the statement. (C) and (D) are incorrect because the information is covered by the attorney-client privilege as well as the work-product privilege. ************************** Next Answer****************************

Sponsor Documents

Or use your account on DocShare.tips

Hide

Forgot your password?

Or register your new account on DocShare.tips

Hide

Lost your password? Please enter your email address. You will receive a link to create a new password.

Back to log-in

Close